Immunology Flashcards
What are the different types of allograft?
Solid organs: kidney, liver, heart, lung, pancreas
Small bowel
Free celss: bone marrow stem cells, pancreas isles
Temporary: blood, skin
Privileged sites: cornea
Framework: bone, cartilage, tendons, nerves
Composite: hands, face
Def: Allograft
Allotransplant (allo- from theGreek meaning “other”) is thetransplantation of cells, tissues, or organs, to a recipient from a genetically non-identical donor of the same species.[1] The transplant is called an allograft, allogeneic transplant, or homograft.
What are te main causes of renal allograft loss?
Infection
Rejection
Obstruction of the uretur
Vascular problems
Recurrent disease in the graft
What are the 3 stages of transplant rejection
Phase 1: recognition of foreign antigens
Phase 2: activation of Ag-specific lymphocytes
Phase 3: effector phase of graft rejection
What are the most relevant protein variations in clinical transplantation that lead to recognition of allograft as foreign?
ABO
HLA (coded on chromosome 6 by MHC)
(minor histocompatibility genes are some other determinants)
What are the 2 major components to rejection
T cell rejection
Ab-mediated rejection: B-cells
What is ABO?
A and B glycoproteins on BCs but also on endothelial lining of BVs in transplanted organ.
There are naturally occuring anti-A and anti-B Abs
What is the A antigen?
N-acetyl-glucosamine
What is the B antigen?
Galactose
AB antigen?
Has both N-acetyl-galactosamine and galactose on glycoproteins
Complete the table


What are HLA?
Cell surface protines
Involved in presentation of forgeign Ags to T-cells
Where are HLA Class I found?
A, B, C expressed on all cells
Where are HLA Class II found?
DR, DQ, DP
Expressed on APC but also upregulated on other cells during stress
What is the importance of HLA in infections/neoplasia vs transplantation?
Maximise diversity in defence against infections, each individual has a variety of HLA, which are derived from a large pool of population varieties.
Variability in HLA molecules in the population provides a source for immnisation against the transplanted organ
Complete the table


HLA MM Parent to child
Sibling to sibling
How many HLA loci?
>3/6 MM
25% 6MM
50% 3MM
25% 0MM
6
HLA Ags in transplantation
Exposure to foreing HLA molecules results in immune reaction to foreign epitopes, this causes damage to the graft-> rejection
T cell mediated rejection in transplantation
Require presentation of foreign HLA by APC, in context of HLA to initiate activation of alloreactive T-cells
What is the difference between the direct and indirect pathways in allograft rejection?
Direct: Donor APC presenting Ag and or MHC to recipient T cells. Acute rejection mainly involves direct rejection.
Indirect: recipient APC presenting donow antigenn to recipient T cells, mainly chronic rejection
T cell vs B cell recognition
T cells recognise Ag with MHC, B cell can recognise just Ag
T cell activation leads to:
Proliferation
Cytokines
Activation of CD8+
Ab production
Recruite phagocytes
Leads to Type IV hypersensitivity
What are the effector cells in T-cell mediated allograft rejection?
Cytotoxic CD8
CD4
Macrophages
What do CD4 cells do in allograft rejection in what phase?
Graft infiltration by alloreactive CD4 cells
Phase 3
What do cytotoxic T cells do in allograft rejection and in what phase?
Release toxins to kill target: Granzyme B
Punch holes in target: perforn
Apoptotic cell death: Fas-L
Phase 3
What do macrophages do in allograft rejection and in what phase?
Phagocytose
Release proteolytic enzymes
Produce cytokines
Produce O radicals and N radicals
What are the symptoms of acte T cell mediated rejection
Deteriorating graft function e.g. RFTs, LFTs, pulmonary oedema dependant on graft type
Pain and tenderness over graft
Fever
A 55 year old man is day14 post deceased donor kidney transplantation. After initial good kidney function with a creatinine falling to 100 umol/L, a routine follow-up finds a creatinine of 145 umol/L
What do you do?
- Ultrasound examination of the graft
- Biopsy of the graft
- Urine analysis
- Surgical exploration of the graft
- Treat blind with corticosteroids
Graft biopsy
What may you see in a graft biopsy of acute cellular rejection of an allograft
Inflammation e.g. tubulitis, arteritis
What occurs in Phase 3 of allograft rejection leading to inflammation?
Abs bind to graft endothelium e.g. capillaries of glomerulus and around tubules, arterial in kidney for e.g.
Net result is vasculopathy
What is the difference between AB Abs and anti-HLA Abs
What can anti-HLA Abs be?
Anti-A/Anti-B are naturally occuring
Anti-HLA Abs are not naturally occuring
Preformed: previous exposure to epitopes e.g. previous transplantation, pregnancy, transfusion
Post-formed: arise post-transplantation
What are the features of chornic rejection of the liver?
Fibrosis
What are the features of chornic rejection of the kidney?
Fibrosis
Glomerulopathy
Capillary BM membrane changes
What are the features of chornic rejection of the pancreas
Fibrosi
Vasculopathy
What are the features of chornic rejection of the lung
Bronchiolitis obliterans
What are the features of chornic rejection of the heart?
Vasculopathy?
What are the types of rejection?
What is thus important?
- T-cell mediated, antibody-mediated or combined
- Importance of graft biopsy for diagnosis as management and outcome are different
How can you prevent rejection?
Treat?
What is important to consider?
AB/HLA matching
Screening for anti-HLA Abs
Immunosuppression
More immunosuppressoin
Balance the need for immunosuppression with the risk of infection/malignancy/ toxicity
When does the screening for Abs in transplantation occur?
Before
At time
After
What are the 3 main types of anti-HLA Ab screening assays?
Cytotoxicity
Flow cytometry
Solid phase
What is the principle for cytotoxicity assay in transplantation?
Does the recipient serum kill the donor’s lymphocytes in the presence of complement
What is the principle for flow cytometry in transplantation?
Does the recipient’s serum bind to the donor’s lymphocytes?
What is the principle for solid phase assays in the context of transplantation?
Does the recipient’s serum bind to recombinant single HLA molecules attached to solid supports such as beads?
Modern transplant immunosuppression outline:
Induction agent: ex. AKT3/ATG, anti-CD52 (alemtuzumab), anti-CD25 (Basiliximab)
Base-line immunosuppression: Calcineurin Inhibitor + mycophenolate mofetil (MMF) or Azathioprine +/- steroids
Treatment of acute rejection: cellular
Steroids. ATG/OKT3
Treatment of acute rejection: Ab-mediated
IVIG, anti-CD5, anti-CD20
PLEX
What is GvHD?
- Eliminate hosts immune system (total body irradiation; cyclophophamide; other drugs)
- Replace with own (autologous) or HLA-matched donor (allogeneic) bone marrow
- Allogeneic HSCT leads to reaction of donor lymphocytes against host tissues
- Related to degree of HLA-incompatibility
Also Gv Tumour effect
What is used for GvHD prophylaxis?
Methotrexate/cyclosporine
Features of GvHD?
Rx
Skin: rash
Gut: N+V, abdo pain, diarrhoea, bloody stool
Liver: jaundice
Corticosteroids
What are the common post-transplantation infections?
What is another consideration post-transplantation in the immunosuppressed?
Generally increased risk for conventional infections.
Opportunistic infections can give sevre infections because of immune compromise: CMV, BK virus, PCP
Malignancy:
Viral associated x100: HHV8, EBV
Skin cancer x20
Risk of other cancers also increased
Transplant reactions: What are the features of hyperacute?
Time
Mechanism
Pathology
Treatment
Mins-Hrs
Preformed Ab which activates complement
Thrombosis and necrosis
Prevention: crossmatch
Transplant reactions: What are the features of acute- cellular
Time
Mechanism
Pathology
Treatment
Weeks-Months
CD4 activating a type IV reaction
Cellular infiltrate
T-cell immunosuppression
Transplant reactions: What are the features of acute- Ab-mediated
Time
Mechanism
Pathology
Treatment
Weeks to months
B-cell activation- Ab attacks vessels
Vasculitis, C4d
Ab-removal and B cell immunosuppression
Transplant reactions: What are the features of chronic
Time
Mechanism
Pathology
Treatment
Mths-Yrs
Immune and non-immune
Fibrosis
Minimise organ damage
Transplant reactions: What are the features of GvHD
Time
Mechanism
Pathology
Treatment
Days- weeks
Donor cells attacking host
Skin, Gut, Liver
Prevention/immunosuppression c corticosteroids
Apart from malignancy and infection, what is another post-transplantation complication?
Why?
Athersclerosis/hyperlipidaemia
x20 increased risk in death from MI compared to age-matched general population
Endothelial activation
Clinical features of immunodeficiency
Infections: 2 major or 1 recurrent minor infection
Unusual: organism or sites
Unresponsiveness to oral Abx
Chronic infections
Early structural damage
Suggestive of priamry:
Failure to thrive
Skin rash
Chronic diarrhoea
Mouth ulceration
FHx
How can immunodeficiencies be classified
Primary: congenital
Secondary
Physiological
What are the secondary causes of immunodeficiency?
Infection: HIV, measles, MTB
Biochemical: malnutrition, DM, renal insufficiency, specific mineral deficiency (Zn, Fe)
Malignancy: myeloma, leukaemia, lyymphoma
Drugs: corticosteroids, anti-proliferative immunosuppressants, cytotoxics
Deficienices of barriers to infection:
Epithelial
Burns victims, high risk of infeciton.
>70% of deaths within 5d of burns relate to infection
Deficienices of barriers to infection:
Mucosal barriers and IgA deficiency
Complete deficiency of IgA affects 1:600 caucasians
Associated with recurrent respiratory and GIT infections in 30%
Deficienices of barriers to infection:
Commensal bacteria
Eradication of normal flora with broad spectrum abx resuts in opportunistic infection:
Candida albicans
C. diff
Phagocyte deficiency:
Recurrent infections in skin and mouh:
Recurrent deep bacterial infections: s. aureus, enteric bacteria, MTB and atypical mycobacteria
Recurrent fungal infections: candida, aspergillus fumigates and flavus
How can phagyocyte deficiencies be characterised?
Recruitment: mobilisation from bone marrow, migration to site of infection
Fight and catch microorganisms
Killing of microogransims
Recruitment of other cells
What is a disorder of phagyocyte mobilisation from bone marrow to within tissues?
Reticular dysgenesis: failure of SC to differentiate along myeloid or lymphoid lineage
Feautres of Reticular dysgenesis
Autosomal recessive severe SCID.
Mutation in AK2 (mitochondrial energy metabolism enzyme)
What is a disorder of failure of neutrophil maturation
Cyclic neutropenia
Kostmann syndrome
What is cyclic neutropenia?
AD episodeic neturopenia every 4-6w due to mutation in neutrophil elastase ELA2
What is Kostmann syndrome?
AR severe congenital neutropenia, classical form due to mutation in HCLS1-associated protein X-1?
What is classically due to mutation in HCLS1-associated protein X-1?
Kostmann syndrome: AR severe congenital neutropenia
What is a disorder of failure of phagocyte migration to site of infection?
Leukocyte adhesion deficiency (CD18 deficiency)
What is CD18
B2 integrin subunit
Features of leukocyte adhesion deficiency?
CD11a/CD18 and CD11b/CD18 are expressed on neutrophils, bind to ligands on endothelial cells and regulat neutrophil adhesion/transmigration.
In leukocyte adhesion deficiency, the lack of these molecules means the neutrophils fail to leave the blood stream.
Characterised by very high neutrophil counts in blood and the absence of pus formation
Characterised by very high neutrophil counts in blood and the absence of pus formation
Leukocyte adhesion deficiency
What is a disorder of a failure to find and catch microothanisms?
Failure of endocytosis and formation of phagolysosome
Complement deficiency and antibody production will result in failure of opsonisation
What is a disorder characterised by failure in oxidative/non oxidative killing?
Chornic ganulomatous disease
Features of chronic granulomatous disease?
Absent respiratory burst: deficiency in one of hte components of NADPHO: inability to generate O free radicals leading to impaired killing of intracellular microorganisms
Excessive inflammation: persistent neutrophil/macrophage accumulation with a failure to degrae antigens leading to granuloma formation.
Lymphadenopathy and hepatosplenomegaly
Ix in chronic granulomatous disease?
Basis is whether neturophils can kill through the production of oxygen radicals- activated neutrophils will stimulate respiratory burst and produce H2O2
NBT
Dihydrorhodamnie flow cytometry
What is NBT?
Nitroblue tetrazolium test- dye will change from yellow to blue when interacted with H2O2:
Chronic granulomatous disease-> no H2O2
What is dihydrorhodamine flow cytometry?
DHR is oxidisaed to rhodamnie which is strongly fluorescent when it interacts with H2O2
No H2O2-> chronic granulomatous disease
What is a disroder of phagocyte failure to recruite other cells?
IL-12/IL-12R
IFNg/IFNgR
Deficiency
Features of IL-12 or IFNg deficiency
Infection with MTB stimulates IL-12 IFNg network
Infected macrophages produce IL-12 which induced T-cells to secrete IFN gamma which feedsback to macrophages and neutrophils to produce TNF, activating NADPHO and stimulating oxidative pathways.
Defect in this pathway leads to susceptibility to mycobacterial infections
What is the approach to treating phagocyte deficiencies?
Aggressive management of infection: infection prohpylaxis: septrin (abx), itraconazole (anti-fungal). Oral/IV Abx as needed. Drainage of abscesses.
Definitive therapy: BM transplantation to replace defective population. Specific treatment fo chronic granulomatous disease= IFNg therapy


What are the clinical features of T cell deficiency
Increased susceptibility to:
Viral infections (CMV)
Fungal (penumocystis, cryptosporidium)
Some bcaterial esp. MTB, salmonella
Early malignancy
What are the clinical features of Ab deficiency or CD4 T cell deficiency?
Bacterial infections (Staph, strep)
Toxins (tetanus, diptheria)
Viral infections: enterovrius
What is reticular dysgenesis?
Most severe form of SCID, mutation of AK2
Failure to produce: neutrophils, lymphocytes, monocytes, macrophages and platelets.
Fatal in early life unless corrected with a BM transplant
How can deficiences of the adaptive immune system be classified?
Defects of haemopoetic stem cells
Defects of lymphoid precuross
Defects in T cell maturation/selection in the thymus
Defects in T cell effector function
What is the defect of haemopoetic stem cells leading to deficiency in the adaptive immune system?
Reticular dysgenesis- SCID
What is the cause of defects in lymphoid precurors leading to deficiency in the adaptive immune system?
Other forms of SCID
Features of SCID
Unwell by 3 months
Infections of all types
FTT, persistent diarrhoea
Unusual skin disease: colonisation of the empty bone marrow by maternal lymphocytes- graft vs host disease.
Fhx of early infant death.
Neonate protected from SCID by maternal IgG in first 3/12
What are the causes of SCID?
20 pathways identified: deficiencies of cytokine Rs, singalling molecules or metabolic defects.
Specific mutation will have a different effect on the lymphocyte subset.
Features of X-linked SCID
45% of all SCID
Mutations of gamma chain of IL-2 receptor on chromosome Xq13.1
Shared by receptros for IL-2, 4, 7, 9, 15 and 21.
Inability to respond to cytokines causes early arrest of T cell and NK cell development and production of immature B cells.
Phenotype: very low or absent T cell numbers
Normal or increased B cell numbers
Poorly developed lymphoid tissue and thymus
Mutations of gamma chain of IL-2 receptor on chromosome Xq13.1
X-linked SCID
What are the causes of T-cell deficiency due to defective maturation/selection in the thymus?
Di George syndrome
Bare lymphocyye syndrome
What are the features of DiGeorge syndrome
Developmental defect of the 3rd and 4th pharyngeal pouch resulting in congenital thymic apalsia.
AD
75% sporadic deletion at 22q11
Phenotype: normal B cell numbers, reduced T cell numbers. Homeostatic proliferation with age and improved function with age
High forhead, low set abnormally folded up ears. Cleft palate. Small mouth and jaw, hypocalcaemia, oesophageal atresia, T cell lymphopenia and complex congenital heart disease
High forhead, low set abnormally folded up ears. Cleft palate. Small mouth and jaw, hypocalcaemia, oesophageal atresia, T cell lymphopenia and complex congenital heart disease
DiGeorge
Deletion at 22q11
DiGeorge

DiGeorge
Features of bare lymphyocyte syndrome Type 2
Defect in one of the regulatory proteins involved in HLA-II expression (regulatory factor X class II transactviator)
No expression of MHCII
Profound deficiency of CD4, usually have normal CD8, normal B cells with a failure to make IgG or IgA Ab.
Type1: failure to express HLAI
Clinical phenoytpe: unwell by 3/12, infections of all types, FTT, can be associated with sclerosing cholangitis.
Fhx of early infant death
What are the defects in T cell effector function leading to immunodeficiency?
Defects in cytokine production, receptor, cytotoxicity or T-B cell communication
e.g. IL-12 IFNg vs MTB
Cytokine R/ secretion deficiency,
Ix in T cell deficiency
Total WCC and differential (NB WCC much higher in children than adults)
Examine lymphocyte subsets: quantify CD8, CD4, DB cells and NK cells
Serum IgG and protein electrophoreisis (IgG prod is a surrogate marker for CD4 celll function)
Functional test of T cel activation and proliferation
HIV test


Mx of T cell deficiencies
Infection prophylaxis
Aggressive treatment of infection
Ig replacement
BM transplant: to replace abnormal population in SCID, to replace abnormal cells
Gene therapy
Thymic transplantation: DiGeorge
Features of Brutonj’s X-linked hypogammaglobulinaemia
Affects B cell maturation
Defective B cell RtK gene
Pre-B cells cannot develop to mature B cells, therefore there is an absence of mature B cells.
No circulating Ig after 3/12
Recurrent infections during childhood, bacterial and enterovirus
Features of selsective IgA deficeincy
2/3rd asymptomatic
1/3rd have recurrent RTI
There is a genetic component, as of yet unidentified
Features of Hyper IgM syndrome
X linked, no IgG, E or A. IgM B cells and plasma cells are present
Clinical phenotype: boy will present in the first few years of life with recurrent infection, bacterial. FTT. Can have PCP, autoimmune disease and malignancy
Normal numbers of circulating B cells and normal T cell numbers with normal T cell in vitro responses.
Elevated IgM with undetectable IgG, IgE and IgA
No germinal centre development within LNs and spleen.
Failure of isotpye swithching
Pathology of Hyper IgM
T cell defect
Mut in CD40 ligand gene. (Member of TNFR, involve in T-B cell communication- expressed by activated T cells)
Failure to express CD40L on activated T cells means T cells are unable to help B cells resulting in no cross talk and failure of Ab switching.
Causes a secondary defect in B celll maturation.
Intrinsic T cell defect
Features of Common Variable Immune Deficiency
Heterogeneous group of disorders with unkown mechanism
Clinical features:
Low IgG, IgA and E
Recurrent bacterial infections, often with severe end organ damge: bronchiectasis, persistent sinusitis, recurrent GI infection
Autoimmune disease
Granulomatous disease
Ix in B cell deficiency
Total WCC and differenital.
Lymphocye subsets
Serum Ig and electrophoreisis.
Functional tests of B cell function:
Specific Ab responses to known pathogens
Measure IgG against teatanus, H influenza b and pneumococcus. If specific Abs are low, immnise and repeat measurement 6-8w later


Mx of B cell deficiency
Aggressive mx of infection
Immunological replacement: derived from pooled plasma of donors, contains IgG Abs to a wide variety of common organisms. Lifelong IgG every 3-4w
BM transplant
Immunisation for selective IgA deficiencies
Features of complement deficiency
Infection: particularly with encapsulated bacteria if the alternative and terminal pathways are involved:
N. meningitides- meningococcus
Step pneumonia
GBS
H. infl
Fhx of infections
SLE if early componenets of the classical pathway are involved
Draw the complment pathway

Features of early calssical pathway deficiencies
C1, C2, C4
Immune complexes fail to activate complement pathways, leads to increased susceptiblity to infection.
Early complement is involved in clearance of apoptotic and necrotic cells-> deficienes result in increased load of self Ags, particularly nuclear components-> SLE
Complement activation normally promotes solubilisation of immune complexes. Deficiencies tresult in deposition of immune complexes which stimulate local inflammation in skin joints.
Types include C1q, C1r, C1s, C2 and C4
What is the most common early complement deficiency
C2
Phenotype of complement deficiency
Almost all patients with C2 have SLE
Usually severe skin disease, also increased incidence of infections
Features of MBL pathway deficiency
30% of individuals are heterozygous for mutant protein
6-10% have no ciruclating MBL
Associated with increased infection in patients who have another cause for immune impairment
Features of alternative pathway deficiencies
Inability to mobilise complement rapidly in response to bacterial infections
Clinical: infections with encapsulated bacteria
Very rare
Features of C3 deficiency
Severe susceptiblity to bacterial infections
Increased risk of developing connective tissue disease
Features of deficiency in terminal pathway of complement
Inability to make MAC
Inability to use complement to lyse encapsulated bacteria
N meningitis
S pneumonia
H infl
Features of secondary complement deficiencies
Active lupus causes persistent production of immune complexes and consequent consumption of complement leading to functional complement deficiency
Nephritic factors are autoAbs direceted against components of the complement pathway
C4NeF: nephritic factor of the classical pathway
C3NeF: nephritic factor of the alternative pathway
Nephritic factors stabilise C3 convertase resulting C3 activation and consumption
Often associated with GN, classically memranoproliferative
Ix of complement deficiency
Quantification of complement pathway: C3 and 4 routinely measured. C1 inhibitor decreased in angioedema
Functional complement tests
CH50 complement test
Classical pathwayq
AP50 complement test
Alternative pathway
Mx of complement deficiency
Vaccination
Prophylactic Abs
Treat infections aggressively
Screen family members
What are the functions of the spleen
Clearance of debris: filtering and phagocytosis of particulate matter. Selective removal of dead and dying cells. Removal of red cell inclusions, bacteria and immune complexes
Immune response to infection: reservoir of lymphocytes, site activation and maturation of B and T cells
What conditions are associated with functional hyposplenism?
Congenital asplenia
Haematological disorders: SCD, thalassaemia major, lhymphoproliferative (HL, NHL, CLL), post BM transplant
Gastro disorders: Coeliac, IBD
Connective tissue disease: SLE and RA
What are the implications of hyposplenism
Increased incidence of infection: risk of severe infection x40, risk of fatal sepsis x17
Strep penumonia
H. infl
Meningococcus
Malaria
Other: capnocytophaga canimorsus (dog bites), babesiousus (protozoal infection, tic bites), E Coli, S aureus, GBS, pseudomonas
Mx hyposplenism?
Immunisation: s penuonia, H influenza, Meningococcus, IFV. Most infections occur during 2 years after splenectomy, risk is lifelong
Antibiotics: low dose prophlyactic penicillin V or erithromycin. Keep broad spectrum antibiotics at home and can be started when symptoms of infection
Medic alert bracelet: no functioning spleen
Take prophylaxis when travelling
Seek attention after animal or tick bites
Antibiotic prophylaxis in hyposplenism
penicillin V or erithromyic for at least 2y after splenectomy
Draw T cell deficiencies

Draw B cell deficiencies



Draw neutrophil deficiency

Where is IgA found?
Mucosal areas, saliva, tears, breast milk
IgE function
Allergy, histamine release from mast cells
IgG significance
Can pass transplacentally
Immature B cells express which Ig?
IgM
Bacterial infections in CGD?
PLACESS
Particular susceptibility to catalase positive organisms:
Pseudomonas
Listeria
Aspergillus
Candida
E coli
Staph aureus
Serratia
Prophylactic treatment of CGD?
Trimethoprim
Itraconazole
IFN
Treatment of cyclic neutropenia
G-CSF
What are the two types of latex allergy
Type 1 hypersensitivity: classical spectrum
Type IV hypersensitvity: contact dermatitis
Features of type 1 hypersensitivity to latex
Acute onset of classcial symptoms, spectrum of severity
Mucosal route associated with more severe reactions.
Occupational exposure
What specific patient groups are at more risk of Type 1 hypersensitvity reaction to latex?
spina bifida, CP, patients undergoing multiple urological procedures, preterm infants, pts with indwelling latex devices
What substances have cross-reactivity to latex?
Avocado
Apricot
Banana
Chestnut
Kiwi
Passion fruit
Papaya
Pear
Pineapple
Dx of Type 1 latex hypersensitivity?
Test specific IgE to latex
Features of Type IV hypersensitivity to atex
Contact dermatitis
Usually affects hands or feet due to glove or rubber footwear and it is mainly due to rubber additvies e.g. thiuram rather than latex itself
24-48h post exposure and characterised by pruritis
Rash is well demarcated and often flaky.
No response to anti-histamines
Dx of type IV hypersensitivity to latex
Patch test: moisten blotting paper with susepcted allergen and tape to healthy area of skin for 24-48h
Eczma will be seen where substance is in contact with skin
Bx will confirm infiltrating T cells and granuloma formation
SPUR
Immune deficiency
Serious
Persistent
Unsual
Recurrent infections
Immunological causes of recurrent meningococcal meningitis
Neurological
Complement deficiency
Ab deficiency
BBB disruption e.g. occult skull #, hydrocephalus
What is CH50
Functional test of the classical complement cascade. All components must be in place to give a positive (normal) result
What is AP50
Functional test of the integrity of the alternative pathweay (bacterial cell wall, properdin, factor B, H I) all components must be in place to give normal result


ANA +ve Result, action
anti-dsDNA
Test complement levels: consumption of complement (i.e. low levels of C3 and C4 suggest active lupus)
Kidney funciton:
Urinalysis
Urine microscopy
Renal biopsy
Features of serum sickness
Caused by exposures to antibodies dervide from animals or some drugs e.g. penicllin based medicines.
Penicllin can bind to cell surface proteins hich can act as a neo-antigen and stimulate a very strong IgG response leading to penicllin.
After sensitisation, subsequent exposures will form immune complexes with the circulating penicllin resulting in increased IgG production. Results in immune complex deposition, complement activation and macrohpage infiltrationa nd neutrophils.
Small vessel vasculitis occurs.
IgG deposition in glomeruli, skin and joints causes renal dysfunction, purpuric rash and arthralgia
Clinical features of serum sickness as a result of penicllin
Fever
Arthralgia
Vasculitic skin rash
Renal function deterioriation
Increasing disorientation
Ix in serum sickness
ESR
CRP
LFTs
all increase
Urine microscopy will show blood and protein
Low complenet (classical pathway)
Specific IgG to penicllin
Biopsy: skin or kidney, macrophages and neutrophil infiltrations
Serum sickness, cause of:
Disorientation
Small vessel vasculitis affecting cerebral vessels may compromise oxygen supply to the brain
Serum sickness, cause of:
Purpura
Leakage from inflamed vessels
Local haemorrhage
Clots
Compromised O2 delivery
Mx of serum sickness
Discontinue penicillin
Corticosteroids to decrease inflammation
Fluid management
Prednisolone
Widely used immunosuppressant
Pegylated IFNa
Effective in some inflammatory disease e.g. Behcets
Azathiorpine
Widely used anti-proliferative immunosuppressnat
Check TPMT status before use
What is the importance of TPMT status?
TPMT is measured in patients who are about to start treatment with thiopurine drugs such as azathioprine. TPMT activity varies in the population and this means that different people require different doses of thiopurine drugs to get the desired therapeutic effects. Guidance in the UK recommends that patients commencing thiopurine drugs have their TPMT status checked before treatment begins. The test identifies individuals at risk of developing severe side effects such as lowering ofblood cell counts and a lowered immune response.
Hydroxycholoroquine
Alters pH and affects Ag presentation/processing
Inhibits production of some cytokines
Upregulates apoptosis/clearance
Allopurinol
Xanthine oxidase inhibitor used in gout
Rituximab
Antibody to CD20 that depletes B cells
IVIG
Effective immunosuppressive agent
Precise mechanisms unclear
Used in SLE, Dmy, Pemphigus
Myclophenolate mofetil
Anti-proliferative immunosuppressant wthi preferntail effect on lymphocytes
Adalimumab
Anti-TNFa Ab
May precipitate cutaenous lupus
Cyclophosphamide
Cytotoxic immunosuppressant
Colchicine
Inhibits neutrophils
Used in gout, Behcets
What is the cause of increased susceptibility to infection in MM?
Suppression of production of normal IgG by the malignant clone results in a functional Ab deficiency= immune paresis
Why does the post-partum presentation of RA occur?
Due to changes in Th cell profiles.
Th2 dominates in pregnancy
Th1 postpartum
How to ensure a good response to vaccination?
Good antigen with a variety of epitopes
Sufficeint dosage
Administration via appropriate route?
What are the best routes for vaccine administration?
SC good uptake and processing
IM ok
IV Ag: taken to spleen
Oral is good locally
Intranasal is good but may lead to an allergic response
Vaccination schedule
2months
5 in 1: diptheria, tetnaus, pertussis, polio, H. influenza
Pneumocccal vaccine
Rotavirus
Men B (new vaccine introduced Sept 2015)
What is the 5 in 1 vaccine?
Diptheria, tetnaus, pertussis, polio, haemophilus
Vaccination schedule
3 months
5 in 1 vaccine, second dose
Men C
Rotavirus, second dose
Vaccination schedule
4 months
5 in 1, third dose
Pneumococcal, second dose
Men B, second dose
Vaccination schedule
1y
Hib/Men C booster
MMR
PCV, third dose
Men B, third dose
Vaccination schedule
2-6y
Children’s flu vaccine
Vaccination schedule
3 years and 4 months
MMR, second dose
4 in 1 pre school booster: diptheria, tetanus, pertussis, polio
Vaccination schedule
12-13y/o
Girls only
HPV-, two injections given between 6m and 2y apart
Vaccination schedule
14y/o
3 in 1 teenage booster: diptheria, tetanus, polio
Men ACWY
Vaccination schedule
>65y/o
Flu annually
Pneumococcal pneumonia vaccine
Vaccination schedule
70y/o
Shingles vaccine
What is the difference between a Th1 and Th2 response?
Th1 is cell mediated: Il-2, IFNg, TNF
Th2 is humoral: IL4, 5 6
Features of IFV vaccine
Protection conferred by antibody to HA rather than the cytotoxic CD8+ cells, which control the viral load
HA: receptor biding and membrane fusion glycoprotein
Response maintenance: begins within 1/52 and can last 6/12 or longer
Features of TB vaccination
Protection conferred by T cell response
Maintenance of response lasts 10-15y
Mantoux Test
Injection of 0.1ml of 5 tuberculin units purified protein derivative. Examine arm after 48-72h
Reaction is an area of unduration around the site.
>10mm= +ve result, implies previous TB exposure or BCG
How can immunological memory be measured?
Presence/amount of IgG
Presence of memory T cells (CD45RA and CD45RO)
either numerical or functional tests
What are the characteristics of immunological memory?
Subsequent exposures are more rapid and aggressive
There are different patterns of the expression of cell surface proteins which allows lymphoid cells access to non-lymphoid tissue
Longevity: memory T cells are maintained without continual antigen by low level prolifeation in response to cytokines like IL2, 7 and 15
What are the CD types for memory T cells?
CCR7+/ CD62L high
Migrate efficiently to peripheral LNs and produce IL-2 (no IFN gamma or perforin)
CCR7 is involved in extravastion
CD62L interacts with molecule on HEV wihich mediates attachment and rolling
What are the CD types ofr effector memory cells?
CCR7- CCR62- Low
Not found in LNs but are in other sites
Produce little IL2 but lots of IFNg and perforin
What are the advantages of live vaccines?
Establishes infection with weak/absent symptoms
Lifelong immunity
Activates all arms of the IR: humoral and local
Immunity to multiple antigens which is more durable due to cross reactivity
What are the disadvantages of live vaccines?
Possible reversion to virulance e.g. vaccine associated paralytic poliomyelitis
Storage problems
Safety issues
Problems in people with immunodeficiency
e.g. of live vaccines?
Yellow fever
MMR
Typhoid
TB
Polio
Vaccinia
E.g. of inactivated vaccines?
IFV
Cholera
Bubonic plaque
Polio (Salk)
Hep A
Pertussis
Rabies
Toxoid vaccines
Diptheria
Tetanus
Components/ subunit vaccines?
Hep B antigen (HbS antigen)
HPV (Caspid)
IFV (HA or NA)
E.g. conjugate vaccine
Hib
Meningococcus
Penumoccocus
Advantages of inactivated/component vaccines
No mutation or reversion
Can be used with immunodeficient
Can lead to elimination of WT virus from community
Easier storage
Low cost
Disadvantages of inactivated/component vaccines?
May have poor immunogenicity
Multiple injections
Adjuvants
Often does not follow normal route of injection
Mechanism of DNA vaccines?
Plasmid containing antigen gene is inserted into a mucsle cell where the Ag is replicated. The gene expressed at the cell surface induces an immune response
Advantages of DNA vaccines
Easy production
Stable
Will ressemble a virally infected cell
Immunogenic
MHCI driven
Disadvantages of DNA vaccines
Plasmid could integrate into host DNA
Autoimmune
What is a depot adjuvant?
Adjuvant which increases the IR without altering its specifcity
Release of the antigen is slowed providing a steady stream of antigen exposure
Freund’s adjuvant
What is Freund’s adjuvant?
Oil based depot adjuvant
Alum as an adjuvant
Ag absorbed into Alum and is slowly released.
Alum also immunogenic in itself as it activates Gr1+ cells which produce IL4 and prime naive B cells.
Alum is the primary adjuvant utilised in humans
What is complete Freund’s adjuvant?
Water in oil emulsion containing MTB cell wall componenets
No longer used clinically
Why are the elderly more likely to die of a vaccine-preventable disease?
Immune senescence: increased frequency of terminally differentiated effector memory T cells in the elderly.
Nutrition: failure to mount an adequate response to some vaccines due to a lack of energy due to poor nutrition
CD45RO T cells=
Memory
CD45 RA T cells=
Effector cells
CPG adjuvant
Unmethylated motife used as an immunostimulatory adjuvant
ISCOMs
Immune stimulating complex used as vaccine adjuvant
Experimental multimeric antigen with built in adjuvant
Human Normal IG
Hep A and Measles
HBIG
Hep B Ig
HRIG
Human rabies Ig
VZIG
Varicella zoster Ig
Paviluzimab
Monoclonal Ab for RSV
Ix in fatigue
FBC
U&E
Creatinine
Calcium
LFTs,
Glucose
Thyroid function testss
Causes of microcytic anaemia
Fe deficiency
Thalassaemia
Anaemia of chronic disease
Fe deficiency
Hb
Serum Fe
TIBC or transferrin
Transferrin saturation
Ferritin
Low
Low
Raised
Low
Low
Ferritin
Ferritin is an ubiquitous intracellular protein that stores iron and releases it in a controlled fashion. The protein is produced by almost all living organisms, including algae, bacteria, higher plants, and animals. In humans, it acts as a buffer against iron deficiency and iron overload.[3] Ferritin is found in most tissues as a cytosolic protein, but small amounts are secreted into the serum where it functions as an iron carrier. Plasma ferritin is also an indirect marker of the total amount of iron stored in the body, hence serum ferritin is used as a diagnostic test for iron deficiency anemia.
TIBC
Total iron-binding capacity (TIBC) or sometimes transferrin iron-binding capacity is a medical laboratory test that measures the blood’s capacity to bind ironwith transferrin.[1] It is performed by drawing blood and measuring the maximum amount of iron that it can carry, which indirectly measures transferrin[2] since transferrin is the most dynamic carrier. TIBC is less expensive than a direct measurement of transferrin.[3][4]
TIBC
Transferrin saturation, abbreviated as TSAT and measured as a percentage, is amedical laboratory value. It is the ratio of serum iron and total iron-binding capacity. Of thetransferrin that is available to bind iron, this value tells a clinician how much serum iron are actually bound. For instance, a value of 15% means that 15% of iron-binding sites of transferrin are being occupied by iron. For an explanation of some clinical situations in which this ratio is important, see Total iron-binding capacity. The three results are usually reported together.
Blood film in coeliac
Hypochromic microcytic cells
Poikilocytes (abnormal shapes)
Anisocytosis (size)
Basophilic stippling (aggregated ribosomal material- also seen in ETOHism, lead poisoning, sideroblastic anaemia and beta thalassaemia)
Target cells: high SA:volume ratio
Blood film in Fe deficiency
Hypochromic and microcytic with anisopokilocytosis
Anaemia of chronic disease
Hb
Serum Fe
TIBC
Transferrin saturation
Ferritin
Low
Low
Normal or low
Normal
Normal or high (acute phase marker)
Thalassaemia
Hb
Serum Fe
TIBC
Transferrin saturation
Ferritin
Normal or low
Normal
Normal
Normal
Normal
Blood film in hyposplenism
Howell-Jolly bodies (nuclear remnants)
Target scells
Causes of Fe deficiency
Poor diet
Blood loss
Malabsorption
Combination of above
Causes of hyposplenism
Absent spleen
Poorly functioning spleen: IBD, coeliacs, sickle cell, SLE
Blood film in megaloblastic anaemia
Hypersegmented neutrophils (problem with DNA synthesis)
Macrocytic
Causes of megaloblastic anaemia
B12 or folate deficiency
(poor diet, malabsorption, pernicious anaemia)
DDx of bowel disease with malabsorption
Coealiac
Crohns
Another dx e.g. menorrhagia, pancreatic disease, bacterial overgrowth, tropical sprue, thyrotoxicosis, post-sx, lymphoma, zollinger-ellison
Ix of coeliac
CRP and ESR
Serological tests: anti-endomysial, anti-ttg
Upper GI endoscopy and distal duodenal biopsy
First line antibody in coeliacs?
Anti-TTG
IgA anti-endomysial
Sensitivity 85-94%
95% specific
Will disappear after several months with adoption of a gluten free diet
Anti-TTG
Coeliacs
90-94% sensitivty
95% specific
Correlates with anti-endomysial
What is an issue with anti-endomysial and anti-TTG
Both IgA
Therefore not useful in IgA deficient patients
Anti-gliadn IgA
57-80% sensitivty, 30-50% specficity
Will persist after adoption of gluten free diet
What is the gold standard for dx of coeliac
Duodenal histology
HLA associations in coeliac?
HLA-DQ2 (90%)
others carry DQ8
Pathophysiology of Coeliac
Peptides from gliadin deamidated by TTG and presented by APC
CD4 cells recognise these peptides which are presented by HLA DQ2 or 8 forming immune complex
CD4 activation leads to secretion of IFNg and IL-15
IL-15 promotes activation of intra-epithelial lymphocytes
IELs kill epithelial cells in a NKG2D dependent manner.
Primed gliadin specific T cells provide help for B cells whose surface receptors.
These produce Abs to gliadin. Other B cells primed for TTG and produce IgAs for tese
Villous atrophy, crypt hyperplasia
Villous height: crypt 3:5
|ncreased IEL
Coeliacs
Histopathology in Coeliacs
Villous atrophy, crypt hyperplasia
Villous height: crypt 3:5
|ncreased IEL

Coeliac
Mx of Coeliacs
Dietary management
Advice re LT compiications
Sources for patient information
What foodstuffs contain gluten
Wheat
Barley ryes
Oats
Gluten free food?
Rice, corned beef, eggs, chips, wine
Cxs of coelacs
Malabsorption
Osteomalacia and osteoporosis
Neurological disease (epilepsy, cerebral calcification)
Lymphom
Hyposplenism
Mortality x2-3 normal population but normalises after 3-5y gluten free
What autoimmune diseases are associated with coeliacs?
Dermatitis herpetiformis (100%)
T1DM (7%)
Autoimmune thyroid disease
DS
SLE, AI hepatitis, AI Addison’s, recurrent apthyous ulceration, Sjogren, sarcoid, vitiligo, alopecia, IgA deficiency
FU Ix in coeliac
Haematology: FBC, Iron studies, Vit B12 and folate, PT time
Biochemistry: U&E, creatinine, Ca and P, LFTs, albumin, total serum protein levels
Serological: quanititaive igA anti-TTG or anti-endomysial
Imaging: DEXA of spine and hip every 3-5y
Indications for testing for coeliacs
Fe def anaemia
Crhonic diarrhoea
Recurrent mouth ulcers
IBS
Chronic fatigue
Unexplained weight loss
FTT
Epilepsy
Peripheral neuropathy
Infertility
FHx of coeliacs
T1DM
DS
AI thyroid disease or vitiligo

Dermatitis herpetiformis
Autoimmune vs autoinflammatory
Autoimmune is an adaptive immune response, can be monogenic or polygenic. Aberran B and T cell responses in primary and secondary lymphoid organs lead to breaking of tolerance and development of immune reactivity towards self-antigens
Autoinflammatory: innate immune responses can be monogenic or polygenic. Local factors at sites predisposed to disease lead to activation of innate immune cells e.g. neutrophils and macrophages resulting in tissue damage
Give 2 examples of monogenic autoinflammatory diseases
Familial Mediterranean fever
TRAPS
What is TRAPS?
TNF receptor associated periodic syndrome
TNRSF1 mutation of TNF receptor.
Periodic fever, rash abdo pain
TNRSF1 mutation of TNF receptor.
Periodic fever, rash abdo pain
TRAPS
What is Familial Mediterranean Fever
MEFV mutation leading to defective pyrin-marenostrin.
AR from people around the mediterranian sea
Sephardic> Ashkenazy Jews, Armenian, Turkish and Turkish
Pyrin marenostrin expressed in neutrophils. There is a failure to regulate cryopyrin driven activation of neutrophils
Periodic fever lasting 48-96 hours
Associated with abdominal pain, arthritis, chest pain, myalgia
LT risk of amyloidosis: nephrotic syndrome and renal failure
Familial mediterranean fever
Rx of Familial Mediterranean Fever
Colchicine 500ug BD
Anakinra: IL-1R antagonist
Etanercept: TNF alpha inhibitor
What are three examples of monogenic autoimmune disease?
APS-1
ALPS
IPEX
What is APS1?
Autoimmune polyendocrine syndrome (Candidiasis-Hypothyroidisim-ADdison’s disease)
AR defect in AIRE (TFactor) involved in T cell immunotolerance in the thymus
Abs vs endocrine tissues
Mild immunodeficiency leads to candida infections
AR defect in AIRE (TFactor) involved in T cell immunotolerance in the thymus
ALPS1
What is IPEX
Immune dysregulation, polyendocrinopathy, enteropathy, X linked syndrome
Mutation in Foxp3 which is required for Treg development.
Overwhelming disease leads to early death without treatment
Usually insulin dependant DM with autoantibodies, thyroid disease, Diarrhoea, eczematous dermatitits
Mutation in Foxp3 which is required for Treg development.
IPEX
What is ALS
Autoimmune lymphoproliferative syndrome
FAS pathway mutations e.g. TNRSF5 which encodes FAS. Leads to defect in Fas-mediated apoptosis. Leading to a chronic non-malignant lymphoproliferation, autoimmune disease and secondary cancers.
Commonly autoimmune cytopenias e.g. AIHA, neutropenia or thrombocytopenia.
High lymphocyte numbers with large spleen and LNs, may be associated with lymphomas
FAS pathway mutations e.g. TNRSF5 which encodes FAS. Leads to defect in Fas-mediated apoptosis. Leading to a chronic non-malignant lymphoproliferation, autoimmune disease and secondary cancers.
Commonly autoimmune cytopenias e.g. AIHA, neutropenia or thrombocytopenia.
High lymphocyte numbers with large spleen and LNs, may be associated with lymphomas
Autoimmune lymphoproliferative syndrome
Give some examples of polygenic autoinflammatory diseases
Crohn’s
UC
Osteoarthritias
GCA
Takayasu’s arthritis.
Features of Crohn’s
IBD1-8 is NOD2 which enodes for CARD-15. mutations in these genes associated with Crohn’s
30% have mutation.
Abdo pain, tenderness, diarrhoea (blood, pus, mucus), fever, malaise oral ulcers, pyoderma gangrenosum, arthritis, raised inflammatory markers
What proportion of Crohn’s patient have NOD2 mutations?
30%
NOD2 CARD-15=
Crohn’s
Features of GCA
Most common systemic vasculitis in the elderly. Affects aorta and cranial branches- temporal artery arising from the external carotid and the opthalmic artery arising from the internal.
Associated with a number of genetic polymorphisms
Temporal headache, claudication pain on chewing, visual loss
Temporal headache, claudication pain on chewing, visual loss
GCA
Ix of GCA
High CRP and ESR
Abnromal temporal biopsy with intimal proliferaiton, disrupted internal elastic lamina and mononuclear cells throughout the vessel wall
Abnromal temporal biopsy with intimal proliferaiton, disrupted internal elastic lamina and mononuclear cells throughout the vessel wall
GCA
What are some examples of mixed pattern auto diseases?
Ank spond
Psoriatic arthritis
Behcet’s
Rx in GCA
High dose corticosteroids.
Immunosuppression LT
Features of ank spond
Inflammatory disease primarily of the axial skeleton
>90% heritability of disease factors.
HLA-B27 there is an environmental trigger: enteric bacteria
Enhanced inflammation at sites of high tensile forces- enthuses: site of instertions of ligaments or tendons.
HLA-B27 associated with?
Ank spond
Lower back pain and stiffness, worse after rest, pain and swelling affecting hips and kness.
Enthesitis
Dactylitis
Uveitis
Ank spond
Rx of ank spond
NSAIDs
Biologic: TNF alpha antagonists
Gives some examples of polygenic autoimmune disease
RA
MG
Pernicious anemia
Addison’s
SLE
PBC
HLA-DR15 associated with
Goodpasture’s
HLA-DR3 associated with?
Grave’s, SKE, T1DM
HLA-DR4 associated with
T1DM, RA
What are the features of central tolerance in T cells
pre-T cells from the BM undergo negative selection for high affinity HLA or neglect for low affinity HLA
What are the features of central tolerance in B cells?
Tolerance occurs in the BM. Self-reactive B cells are deleted. Immature B cells are deleted by polyvalent antigens.
Only the immature B cells that have no cross-linking survive
What are the mechanisms of peripheral tolerance?
Anergy
Regulatroy Cells
Immune privilege
What is anergy
Lack of co-stiumlating molecules means T cells will not respond to subsequent challenge. CD40/CD40L. CD800/86-28
What is the mechanism for peripheral tolerance through regulatory cells
Decreased number of regulatory cells can lead to AID
T regs: IPEC
Tr1 cells= IL10 secreting
CD8 regulatory cells
How does immune privilege lead to peripheral tolerance
Lymphocytes are denied enry e.g. eye, CNS, testes.
Damage to these sites can cause AID
What is the Gel-Coombs classification
Relates to the effectory mechanism of tissue damage rather than the autoimmune repsonse.
What is Type I hypersensitivity
e.g.
Immediate hypersenstivity which is mediated by IgE
e.g. atopy, anaphylaxis, asthma
What are the features of a Type I hypersensitivity reaction?
Rapid allegic reaciton when the allergen binds to preformed antibodies.
Antgen binding leads to cross linking and mast cell degranulation
This leads to the release of preformed inflammatory mediators e.g. histamine, serotonin, proteases as well as a delayed reaction due to synthesised mediators e.g. leukotrienes, prostaglandins, bradykinin and cytokines.
Leads to incresaed vascular permeability, smooth muscle contraciton and leukocyte chemotaxis
What is Type II hypersensitivity?
Cytotoxic HS. Antibody reacts with cellular antigen.
e.g. pemphigus vulgaris: epidermal cadherin causes skin blisering.
Goodpastures: non collagenous daamage of BM
AIHA: Rh blood group anitgen.
Often causes autoimmunity
What are the featurse of type II hypersensitvity
Auto-ab binds to a cell or amtrix associated antigen on the affected organ cells. Results in antibody dependant cell dest5rcution via complement, NK cells or phagocytes
Complenet: classical pathway
NK: release of cytotoxic granules
Phagocytes: phagotcytosis
What is a Type V HS response
Receptor activation or blockaed.
Instead of binding to cell surface components as in a Type II reaction, Abs recognise and bind to cell surface receptros which either prevent the intended ligand binding to the receptor or mimic the ligand e.g. Grave’s. MG
What is a type III hypersensitivity reaciton?
Immune complex-HA
e.g. cryoglobulinaemia.
SLE
Serum sickness
Features of a type III hypersensitivity raction
Antibody reacts with a soluble antigen to form an immune complex.
Immune complex deposited in BVs which can activate complement and initiate macrophage and neutrophil infiltration.
Overall result is inflamamtion and damage to the vessels.
Symptoms include fever, vasculitis, arthritis, nephrtiis.
Usually immune complexes are cleared but they persist in autoimmune disease
What is a type IV hypersensitivity response?
e.g.
Delayed- T cell mediated
T1Dm
MS
Allergic contact dermatitis
Psoriarsis
Features of Type IV hypersensitivity
MHC1 cells present a self-pepdite which is recongised by CD8 cells, whic perform cytotoxic function leading to cell death.
MHCII present self-antigens to the CD4 cells. Along with a costimulatory signal, this results in IFNg release activating macrophages. This results in tissue damage.
The Th1 CD4 cells need to have been previously primed to react to the self-antigen.
Pathophysiology of Type I Hypersensitivity
Immediate reaciton as a response to re-exposure to an allergen
IgE mediated: mast cells release mediators resulting in vasodilation, increased permeability and smooth muscle spasm
Typica symptoms of a T1H reaction
Angioedema
Urticaria
Rhinoconjunctivitis
Wheeze
D+V
Anaphylaxis
Atopy=
Produciton of a specific IgE response to common environmental allergy
Allregy=
Development of a T1H to an environmental allergen
Triad of atopy?
Eczema
Asthma
Hay fever
Allergy onset
Infancy
Atopic dermatitis
Food allergy (milk, eggs, nuts)
Allergy onset
Childhood
Asthma
Allergic rhinitis
Allergy onset
Adults
Drug
Bee
Oral allergy syndrome
Occupational allergy
Pathology in atopic dermatitis?
Defects in beta defensin predisposing to staph aureus superinfection
Irritants in atopic dermatitis
Irritants, food, environmental
Treatment of atopic dermatitis
Emollients
Skin oils
Topical steroids
PUVA
Phototherapy
IgE mediated food allergy
Anaphylaxis
Cell-mediated food allergy
Coeliac
IgE/cell-mediated allergy
Atopic dermatitis
Dx of food allergy
Food diary
Skin prick tests
RAST
Challenge tests
Resolve by adulthood
Mx of food allergy
Dietician, food avoidance, epipen
Control asthma if present
Common allergens in food allergy?
Milk
Egg
Peanut
Shellfish
Fish
Tree nut
Oral allergy snydrome
Exposure to allergen induces food allergy
Symptoms limited to mouth;
2% get anaphylaxis
Allergens in OAS
Birch pollen + Rosacea fruit
Ragweed +Melons
Mugwort + celery
Mx of OAS
Avoid food
If ingested wash mouth and take antihistamine
Allergens in latex food syndrome
Chestnut
Avocado
Banana
Potato
Tomato
Kiwi
Papaya
Eggplant
Mango
Wheat
Melon
Pathology in latex food syndrome
Some foods have latex like components and hence latex allergy sufferers will also have food allergies
Dx of LFS
Skin prick
Allergens in allergic rhinitis?
Seasonal (tree and grass pollen, fungal spores)
Perennial (pets, house dust mite)
Occupational (latex, lab animals)
Symptoms of allergic rhinitis
Nasal itch and obstruction
Sneezing
Anosmia
Eye symptoms
RAST test
A radioallergosorbent test (RAST) is a blood test used to determine the substances a subject is allergic to. This is different from a skin allergy test, which determines allergy by the reaction of a person’s skin to different substances.
Dx of allergic rhinitis
Pale bluish swollen nasal mucosa
Skin prick
RAST
Mx of allergic rhinitis?
Allergen avoidance
Anti-histamine
Steroid nasal spray
Sodium cromoglycate eye drops
Oral steroids
Ipratropium nasal spray
Grass pollen desensitisation
Allergen in acute urticaria?
50% idiopathic
50% caused by food, drugs, latex, viral infectyions and febrile illnesses
Pathology of acute urticaria
IgE-mediated reaciton
Wheals which resolve within 6w
Def: anaphylaxis
A severe systemic alelrgic reaciton: respiratory difficulty and hypotension
What are the mechanisms of anaphylaxis?
Can be IgE mediated: peanut, penicllin, wasp or bee venom, latex
Or
Non-IgE mediated: aspirin and NSAIDs, IV contrast media, opiod analgesia, exercise
Dx of anaphylaxis
Serum typtase
Concentration proportional to fall in BP, rises to peak 60 mins after exposure
Mx of anaphylaxis
ABC:
Lift legs to boost venous return
100% O2
Inhaled bronchodilators: salbutaoml
IM adrenaline 500mg
IV hydrocortisone 100mg
IV Chlorphenamine 10mg
IV flyuds
Seek help
Clinical picture in IgE allergic response
Angioedema, urticarial, rhinoconjuncitvitis, wheeze, D+V and vomiting
Clinical picture in non-IgE mediated response
Recurrent abdo pain
Diarrhoea
Fatigue
Migraine
Hyperactivity
Depressoin
Confusion
Skin prick test=
Gold standard
Local wheal
Positive control- histamine
Negative cointrol- diluent
No antihistamines in 48h before hand
Diagnostic tests in allergy
Skin prick
RAST
Component allergen specific IgE
Challenge test
Features of challenge test
Challenge test: medically supervised exposure. Gold standard for food allergy diagnosis. Increasing volumes of the
allergen are ingested. Double blind placebo or open challenge. Expensive in terms of staff clinical time. Risk of severe
reaction
What is component allergen specific IgE testing?
Component Allergen Specific IgE: measures response to a specific allergen related protein. E.g. peanut has 5 major
allergens. Useful for clinical information to severity if given a whole peanut.
Peanut: Ara h 2: anaphylaxis risk to peanut and nuts
Ara h 8: localised oral reaction to peanuts and stone fruits only.
Ara H2
Anaphylaxis risk to peanuts and nuts
Ara H8
Localised oral reaciton to peanuts and stone fruits onlyq
What is an important risk factor for atopic dermatitis?
Filaggrin mutation
Fillagrin
Structural protein which maintains the integrity of the epithelial barrier and contributes to skin hydration
LOF mutation found in atopic dermatitis.
Immunosuppressant treatment options for atopic dermatitis
Topical steroids
Cyclosporin
Tacrolimus
What are the different types of food “allergy”
Intolerance
Aversion
Allergy: IgE mediated
AEG?
Allergic eosinophilic gastroenteritis
IgE/cell-mediated food allergy
Pathophysiology of chronic urticaria
>6w persistent itchy wheals
Idiopathic or autoimmune (IgG against Fc epislone R1 or IgG against IgE) or phyzical
Mx of chronic urticaria
Doxepin: TCA and anxiolytic
Ciclosporin for refractory cases
Pathophysiology of Hereditary angioedema
C1 inhibitor deficiency (C1 inhibitor inhibits the complement systm)
Deficiency of C1 inhibitor allows plasma kallikrein activaiton leading to the production of bradykinin
Symptoms of Hereditary angioedema
Angioedema can occur with mild trauma or spontaneously
Considerations for hereditary angioedema
Can predispose to SLE due to consumptive effect on complement C3 and 4
What are the constitutive barriesr to infection?
Skin
Mucosal surfaces
Commensal bacteria
What are the features of skin that provide a barrier to infection
Tightly packed keratinised cells
Low pH and O2 tension
Sebaceous glands which produce: hydrophobic oils that repel MO and water
Lysozyme which destroys the structural integrity of the cell wall
Ammonia and defnesins which have antibacterial properties
What features of the mucosal surfaces provide a barrier to infection
Cilia that trap and remove pathogens
Secreted mucous:
Secretory IgA
Lysozyme
Lactoferrins: starve Mo of iron
What are the cells of the innate immun system
Polymorphoneuclear:
Neutrophils, eosinophils, basophils
Monocytes and macrophages
NK cells
Dendritic cells
What are the soluble components of the innate immune system?
Complement, acute phase proteins, cytokines, chemokines
What is the principle difference between monocytes and polymorphonuclear cells in terms of PRR?
Can both generically recognise PP but can also present processed antigens to T cells
Macrophage in
Liver
Kuppfer cell
Macrophage in
Kidney
Messangial cell
Macrophage in
Bone
Osteoclast
Macrophage in
Spleen
Sinusoidal lining cell
Macrophage in
Lung
Alveolar macrophage
Macrophage in
Neuronal tissue
Microglia
Macrophage in
Connective tissue
Histiocyte
Macrophage in
Skin
Langerhans
Macrophage in
Joints
Macrophage like synoviocyte
What are opsonins
Facilitate phagocytosis by acting as a bridge between the pathogen and phagocyte receptors
Abs
Complement components
Acute phase proteins
What are the mechanisms of phagocytosis related microbial killing?
Oxidative killing: NADPH oxidase-> ROS
Non-oxidative: release of bacteriocidal enzymes: lysozyme and lactoferrin
Why do phagocytes die after phagocytosis?
Process of phagocytosis depletes glyocgen reserves and is usually followed by neutrophil death
Pus forms from dead neutrophils
What is the predominant cell type in synovial fluid taken from patients with gout?
Neutrophils
Function of NK cells
Important in virally infected cells, malignancy and also activing dendritic cells to promote adaptive immune response.
What is the mechanism to prevent inappropriate activation of NK cells?
Express inhbitory receptors for Self-HLA (Class I)
These are downregulated if the cell is infected
What is the function and mechanism of dendritic cells?
Primes the adaptive immune response
Capable of phagocytosis which results in their maturation, upregulation of HLA molecules, expression of costimulatory molecules and migration to LNs.
Present processed Ag to T cells in LNs to prime AIR and also secrete cytokines
What mediates dendritic cell migration to LNs?
CCR7
What are the components of the adaptive immune system
Humoral: B lymphocytes and antiboides
Cellular: T lymphocytes: CD4 and CD8
Soluble components: cytokines and chemokines
What are the primary lymphoid organs and their function
Bone marrow: B cell maturation
Thymus: T cell maturation
Def: secondary lymphoid organ
Anatomical sites of interaction between naive lymphocytes and Ag e.g spleen, LNs and MALT
Process of T cell maturation?
HSC begin to mature in BM
Undergo TcR rearrangement in which alpha and beta chain segments are rearranged in a semi-random manner
Migrate to thymus as pre-T cells where they undergo selection for CD4 and CD8
Low affinity for HLA-not selected
Medium affinity- positive selection
High affinity: negative selection to avoid autoreactivity (central tolearnce)
Medium affinity for HLA class 1?
CD8+ cells
Medium affinity for HLA Class II
CD4+
Function of CD4 cells
Recognise peptides presented on HLA class II molecules
Immunoregulatory via cell:cell interaciton and cytokine expression
Polarising factors in Th1
IL-12, IFNg
Effector profile of Th1
IL-2 and 10
IFNg
TNFa
Lymphotoxin
Function of Th1
Helps CD8 T cells and macrophages
Polarising factors for Th17
IL-6
TGFb
Effector profile of TH17
IL-17
21 and 22
Function of Th17?
Helps neutrophil recruitment
Enhances the generation of autoantibodies (involved in autoimmune disease)
Treg polarising factors
TGF beta
T reg effector profile
Il-10
Foxp3
CD25
Function of Treg cells
Negative T cell regulators
TFh=
Follicular helper cells
Function of TFh?
Help in promoting germinal centre reactions and differentiation of B cells into IgG and IgA secreting plasma cells
Polarising factors for TFh
Il-6 and 1b
TNFa
Effector profile for TFh?
Il-2 and 10 and 21
Polarising factors for Th2
IL4 and 6
Effector profile for Th2
IL4 and 5 and 10 and 13
Function of Th2
Helper T cells
What is isotype switiching?
CD4 cells are required for process of isotype switiching and differentiation of B cells into IgG and IgA secreting plasma cells, without CD4 cell function the B cell response will be restricted to IgM
Killing mechanism of CD8 cells
Perforin and granzymes
Apoptosis mediation through FASL expression
Process of B cell maturation
Arise from HSCs
Created through semi-random gene rearrangement to produce a diverse receptor repertoire
Central tolerance occurs by deleting high affinity self Rs
B cell activation following antigen encounter
Results in early IgM response
CD4 cells help B cell differentiation through CD40L:CD40 interaction
Leads to somatic hypermutation and isotype switching (switching of heavy chains)
Generates B cells capable of secreting IgG and A and E
What part of the antibody determines the class?
Heavy chain
What portion of the antibody recognises the antigen?
Fab
What determines Ab effector function?
Determined by constant region of heavy chain
Fc
Components of classical component pathway
C1, C4 and C2: Antibody antigen immune complexes
Results in antibody shape change and exposes C1 binding site
C1 activates hte cascade
Depends on the activation of the AIS therefore is in late response
Components of the lectin complement pathway
Lectin: mannose binding lectin binds to oligosaccharides/ microbial cell
surface carbohydrates on certain virions/infected cells. This then directly
stimulates C4 & C2 but not C1. N.B not dependent on the acquired
immune response.
Components of the alternative complement pathway
Alternative: spontaneous breakdown of C3 in serum forming alternative
C3 convertase due to it binding to components in the bacterial cell wall e.g.
lipopolysaccharide of gram –ve, teichoic acid of gram +ve. N.B not
dependent on the acquired immune response. Involves factors B, I & P.
What are the ligands for CCR7?
CCl19 and CCl21
Functions of activated complement?
Increases vascular permeability and cell trafficking
Opsonisation of immune complexes so they remain soluble
MAC: holes in membranes
Promote basophil and mast cell degranulation
Opsonisation of pathogens: esp. capsulated bacteria
Activate phagocytes
In the immature form these cells are adapted for recognition and uptake of pathogens. Maturation is associated with expression of CCR7, migration to lymph nodes and enhanced capacity for antigen presentation.
- OPTION LIST
- Th17 cell
- Macrophage
- Epithelial cell
- T reg cell
- Dendritic cell
- CD4+ T cell
- Neutrophil
- Th1 cell
- Plasma cell
- Megakaryocyte
- Lymphocyte
Dendritic cell
These cells may be formed following a germinal centre reaction involving isotype switching and affinity maturation of receptors. They are long-lived and reside in bone marrow.
- OPTION LIST
- Th17 cell
- Macrophage
- Epithelial cell
- T reg cell
- Dendritic cell
- CD4+ T cell
- Neutrophil
- Th1 cell
- Plasma cell
- Megakaryocyte
- Lymphocyte
Plasma
C1
A. Binding of immune complexes to this protein triggers the classical pathway of complement activation
B. Cleavage of this protein may be triggered via the classical, MBL or alternative pathways
C. Binds to microbial surface carbohydrates to activate the complement cascade in an immune complex independent manner
D. Part of the final common pathway resulting in the generation of the membrane attack complex
A. Binding of immune complexes to this protein triggers the classical pathway of complement activation
C9
A. Binding of immune complexes to this protein triggers the classical pathway of complement activation
B. Cleavage of this protein may be triggered via the classical, MBL or alternative pathways
C. Binds to microbial surface carbohydrates to activate the complement cascade in an immune complex independent manner
D. Part of the final common pathway resulting in the generation of the membrane attack complex
D. Part of the final common pathway resulting in the generation of the membrane attack complex
What are T2HRs?
IgG or IgM antibody reacts with cell or matrix associated self antigen.
Results in tissue damage, receptor blockade/activation
What type of hypersensitivity reaction is
HDN?
T2
What type of hypersensitivity reaction is
AIHA
T2HS
What is Evan’s syndrome?
Evans syndrome is a very rare autoimmune disorder in which the immune system destroys the body’s red blood cells, white blood cells and/or platelets. Affected people often experience thrombocytopenia (too few platelets) and Coombs’ positive hemolytic anemia (premature destruction of red blood cells)
What type of hypersensitivity reaction is
AITP?
T2HS
What type of hypersensitivity reaction is
Goodpasture’s?
T2H
Goodpasture’s syndrome
Abs vs GBM
What type of hypersensitivity reaction is
Pemphigus Vulgaris?
T2HR
What type of hypersensitivity reaction is
Grave’s disease?
T2HR
What type of hypersensitivity reaction is
MG?
T2HR
What type of hypersensitivity reaction is
Acute Rheumatic Fever?
T2HR
What type of hypersensitivity reaction is
Churg Strauss?
T2HR
What is eGPA?
Churg Strauss
What is GPA?
Wegener’s
What type of hypersensitivity reaction is
Wegener’s?
T2HR
What type of hypersensitivity reaction is
Microscopic polyangitis?
T2HR
What type of hypersensitivity reaction is
Chronic Urticaria?
T2HR
Antigens on neonatal erythrocytes?
HDN
Blood autoantigens eg. Rhesus
AIHA
Antigen=
Glycoprotein IIb/IIIa on platelets
AITP
Antigen=
Noncollagenous domain of basement membrane colagen IV?
Goodpastures
Antigen=
Epidermal cadherin
Pemphigus vulgaris
Antigen=
TSHR
Grave’s
Antigen=
AChR?
Myasthenia Gravis
Antigen=
M proteins on GAS?
Rheumatic fever
Antigen=
IF and gastric parietal cells
Pernicious anaemia
Antigen=
Medium and small vessels
against myloperoxidase with granulomas, eosinophils granulocytes
Churg Strauss
Antigen=
Medium and small vessels
Against Proteinase 3?
Wegener’s
Antigen=
Pauci-immune necrotitsing small vessel vasculitis
Ab against small vessel vasculitis
Microscopic polyangitis
What differentiates between MPA and Churg Strauss
Both pANCA positive
Churg Strauss featues eosinophils
Maternal IgG mediated reticulocytosis and anaemia
HDN
Destruction of RBCs by autoantibodies + complement + FcR + phagocytes
Anaemia
AIHA
Pupura
T2HS
AITP
Glomerulonephritis
Pulmonary Haemorrhage
Goodpastures
Non-tense blistering of the skin and bullae
Pemphigus Vulgaris
Autoimmune hyperthyroidism
Grave’s
Fatiguable muscle weakness
Double vision
MG
Myocarditis
Arthritis
Sydenham’s Chorea
Acute rheumatic fever
Anaemia
Reduced B12
Pernicious anaemia
Allergy
Asthma
Systemic disease
Male predominance
Churg Strauss
Sinus Problems
Lung cavitations and haemorrhage
Crescenteric GN
Wegener’s
Pupura, livedo
Multiple organs affected?
Microscopic polyangitis
Dx HDN
Positive direct
Coomb’s test
Dx AIHA
Positive direct Coombs test
Anti Red Cell Ab
Antiplatelet Ab
AITP
Anti GBM Ab
Linear smooth IF staingin of IgG deposits on GBM
Goodpastures
Direct immunofluorescence of epidermis showing IgG?
Pemphigus
Anti-TSHR Ab
Grave’s
Anti-AChR Ab
Abnormal EMG
Tensilon test
MG
Tensilon test
Acetylcholine is a neurotransmitter chemical that nerve cells release to stimulate your muscles.
People with a chronic disease called myasthenia gravis don’t have normal reactions to acetylcholine. Antibodies attack their receptors for acetylcholine. This prevents muscles from being stimulated and makes muscles tired.
The Tensilon test uses the drug Tensilon (edrophonium) to diagnose myasthenia gravis.Tensilon prevents the breaking down of the chemical acetylcholine, which then helps stimulate the muscles. A person tests positive for myasthenia gravis if their muscles get stronger after being injected with Tensilon.
Jones Criteria
Acute Rheumatic fever
Anti Gastric Parietal Cell Ab
Anti-IF Ab
Schilling Test
Pernicious anaemia
pANCA with eosniophilic granulomas?
Churg Strauss
cANCA
Wegener’s
pANCA
Without eosinophils
Microscopic polyangitis
Mx HDN
Maternal PLEX
Exchange transfusion
Mx AIIHA
Steroids
Mx AITP
Steroids
IVIG
Anti-D Ab
Splenectomy
Mx Goodpasture’s
Corticosteroids and immunosuppression
Mx Pemphigus
Corticosteroids and immunosuppresion
Mx Graves
Carbimazole
Propylthiouracil
Mx MG
Neostigmine
Pyridiostigmine
If serious use IVIG and plasmaphoreisis
Mx acute rheumatic fever
Aspirin
Steroids
Penicllin
Mx Pernicious anaemia
Dietary B12 or IM B12
Mx Churg Strauss
Prednisolone
Azathioprine
Cyclophosphamide
Mx Wegeners
Corticosteroids
Cyclophosphamide
Cotrimoxazole
Mx MPA
Prednisolone
Cyclophopshamide or azathioprine
Plasphoreisis
Antigens in chronic urticaria
Medications (NSAIDs)
Cold
Food
Pressure
Sun
Exercise
Idiopathic
Insect bites
Persistent itchy wheals lasting >6w
Associated with angioedema in 50%
Chronic urticaria
Chronic urticaria responding poorly to anti-histamine
Exclude Urticarial vasculitis
Dx of Urticaria
Challenge test
ESR (Raised in urticarial vasculitis)
Skin prick testing
Mx Chronic urticaria
Avoid precipitants
Check for thyroid disease
Preventative antihistamine
IM adrenaline for pharyngeal angiodema
1% menthol aqueous cream for pruritis
Doxepin and ciclosporin
What type of hypersensitivity reaction is
Mixed essential cryoglobulinaemia?
T3HR
What type of hypersensitivity reaction is
Serum sickness
T3HR
What type of hypersensitivity reaction is
Polyarteritis Nodosa
T3
What type of hypersensitivity reaction is
SLE
T3
Antigen in
Mixed essnetial cryoglobulinaemia
IgM against IgG +/- Hep C antigens
Antigen in
Reaction to proteins in antiserum
Serum sickness
Antigen=
HepB, C virus Ags
PAN
Antigen=
Mainly intracellular components: DNA, histones, RNP
SLE
Joint pain
Splenomegaly
Skin nerve and kidney involvement
Associated with Hep C
Mixed essential cryoglobulinaemia
Rashesm itching, arthralgia, lymphadenopathy, fever, malaise
Developing after 7-12d
Serum sickness
Fever
Fatigue
Weakness
Arthralgia
Skin, nerve and kidney involvement
Pericarditis and MI
Assocaited with Hep B
PAN
SOAP BRAIN MD (>4)
SLE
Dx of mixed essential cryoglobulinaemia
Mixture of clinical and biopsies
Dx of serum sickness
Reduced C3
Blood shows immune complexes or signs of blood vessel inflammation
Dx PAN
Clinical criteria and biopsy
Raised ESR
Raised WCC
Raised CRP
Rosary sign
Rosary sign
The rosary sign is a CT finding in adenomyomatosis of the gallbladder. It is formed by the enhanced proliferative mucosal epithelium, with the intramural diverticula surrounded by the unenhanced hypertrophied muscle coat of the gallbladder. The rosary sign is similar to the pearl necklace sign.
Dx of SLE
Reduced C4 (C3 only reduced in severe disease)
ANA
Raised ESR with NORMAL CRP
What drugs can cause drug-induced SLE
Hydralazine
Procainamide
Isoniazid
What is a T3HR?
IgG or IgM immune complex mediated tissue damage (against soluble antigens)
What differentiates between a T2HR and a T3HR
T2 is Abs against cell matrix associated
T3 is against soluble antigens
Mx of mixed essential cryoglobulinaemia?
NSAIDs
Corticosteroids
Plasmapharesis
Mx of serum sickness
Discontinuation of precipitant
Steroids
Antihistamines
Analgesia
Mx PAN
Prednisolone and cyclophosphamide
Mx SLE
Analgesia and Steroids
Cyclophosphamide
What is a T4HR?
Delayed hypersensitvity
T cell mediated
What type of hypersensitvity reaciton is
T1DM
T4
What type of hypersensitvity reaciton is
MS
T4
What type of hypersensitvity reaciton is
RA
T4
Also T3: IgM Ab vs Fc region of IgG
What type of hypersensitvity reaciton is
Contact dermatitis
T4
What type of hypersensitvity reaciton is
Mantoux
T4
What type of hypersensitvity reaciton is
Crohn’s
T4
Antigen=
Pancreatic beta cell proteins (Glutamate Decarboxylase GAD)
T1DM
Antigen=
Oligodendrocyte proteins (myelin basci protein, proteolipid protein)
MS
Antigen=
Ag in synovial membrane
RA
Antigen=
Environmental cehmicals
Posion ivy
Nickel
Contact dermatitis
Insulitis
Beta cell destruction
T1DM
Demyelinating disease
Perivascular inflammation
Paralysis
Ocular lesions
MS
Chronic arthritis
Nodules
Lung fibrosis
RA
Dermatitis with short lived itching, blisters, wheals
Contact dermatitis
NOD2 mutations in 30% of
Crohn’s
TH1 mediated chronic inflammation in skip lesions in GIT
Crohn’s
Dx T1DM
Blood glucose
Ketonuria
Glutatmate decarboxylase Abs
Islet cell Abs
Oligoclonal bands of IgG on electrophoreisis?
MS
What is the most specific antibody for RA?
Anti-CCP
How sensitive is RF for RA?
85%
Dx of RA
XR
RF
Anti-CCP
ESR
CRP
Dx of crohn’s
Bx of lesion
Mx of T1DM
Injectable insulin or infusion
Mx of MS
Corticosteroids, IFNb
Mx RA
Analgesia
Steroids
DMARDs
Mx non-resolving contact dermatitis
Corticosteroids or antihistamines
Mx Crohn’s
Antibiotics
Mesalazine
Infliximab
Steroids
HLAB27
Ank Spond
HLADR15/DR2
Goodpastures
HLA DR3
SLE
Grave’s
HLADR4
RA
HLADR3/DR4
T1DM
What is PTPN22
Tryosine phosphatase expressed in lymphocytes
Assoc with development of SLE, RA, T1DM
What is CTLA4
R for CD80/86 expressed by T cells
Transmits inhibitory signal to control T cell acitavtion
Assocaited with SLE, T1DM and autoimmune thyroid disease
Anti-centromere Ab
CREST
CREST + GIT + interstitial pulmonary disease + renal problems
Diffuse scleroderm
Anti-SCl70 (topoisomerase)
Diffuse scleroderma
Xerostomia
Keratoconjuncitivitis Sicca
Nose and skin
May affect kidneys, BVs, lungs, pancreas, PNS
May get parotid or salivary gland enlargement
Sjogrens
Anti-Ro and Anti-La
Sjogren’s
Immune dysregulation, polyendocrinopathy, enteropathy with X linked inheritance and autoimmune disease:
ezemtatous dermatitis, nail dystrophy and autoimmune skin conditions
IPEX
IgA anti-endomysial in coeliac
Disapppers with exclusion of gluten
DQ2 or DQ8
2 8 or not to eat
Coeliac
Anticardiolipin
Anti B2 glycoprotein
Lupus anticoagulant
Hughes Syndrome (antiphospholipid)
Anti-SMK
AntiLMK1
Anti-SLA
Autoimmune hepatitis
Anti-Rh blood group antigen
AIHA
Anti-glycoprotein IIb-IIIa
AITP
pANCA
Wegeners
MPA
Anti-TTG
Anti-endomyseal
Coeliac
Anti-Ro
Congenital heart block in infants of mothers with SLE
What causes Congenital heart block in infants of mothers with SLE
Anti-Ro
Antibody in dermatitis herpetiformis
Anti-endomysial antibody
Anti-Jo-1
Dermatomyositis
Polymyositis
Anti-topoisomerase
Anti RNA Pol I,II,III
Fibrillarin
Diffuse scleroderma
Anti-centromere
CREST
Anti-U1RNP Ab
(speckled pattern)
Mixed connective tissue disease
Anti-mitochondrial Ab
PBC
Anti-CCP
RA
Anti Ro
Anti La
70% RF positive
Sjogren
Anti-dsDNA + histones
and Ro La, Sm, U1RNP
SLE
Anti-GAD
T1DM
What are the 4 ENAs?
Ro, La, SM and U1RNP
What type of hypersnseisitivty reaction is
Hashimoto’s thyroidits?
T2 and T4
What T cell class is implicated in T1DM
CD8
SCAD
B12 deficiency
Drooping eyelid worse on repetition and at the end of the day
MG
Impact of environment in RA
Smoking associated with increased citrullination and development of erosive disease
Gum disease also associated
What class of Ab is RF?
IgM
What type of HSR is RA?
Type II: antibodies binding to citrullinated proteins which leads to complement activation, macrophage
activation via Fc R and CR, and NK cell activation with ADCC.
Type III: immune complex formation with rheumatoid factor and anti-CCP leading to tissue deposition and
complement activation.
Type IV: presentation of peptide e.g. type II collagen, chondrocyte gp39, citrullinated peptides by APC, activating
CD4+ T-cell activating macrophages and fibroblasts leading to increased production of MMPs, IL-1 and TNF
alpha.
Treatment of RA
First line:
DMARD:
methotrexate, sulphasalazine, hydoxychlorowuine and leflunomide if methotrexate not tolerated
TNF antagonists:
Rixumiab
Atabacept
Tocilizumab
What are the anti-nuclear Ab connective tissue diseases?
SLE
Systemic sclerosis
Sjogrens
Dermato/polymoysitis
What must be excluded in inflammatory myositis
Presence of underlying malignancy
Anti-SRP
Polymyositis
•Recurrent infections with hepatosplenomegaly and abnormal dihydrorhodamine test
A. IFN gamma receptor deficiency
B. Leukocyte adhesion deficiency
C. Chronic granulomatous disease
D. Kostmann syndrome
CGD
•Recurrent infections in childhood, abnormal facial features, congenital heart disease, normal B cells, low T cells, low IgA and IgG
A. Bare lymphocyte syndrome type II
B. X-linked SCID
C. DiGeorge syndrome
D. IFN gamma receptor deficiency
Di-George
•1 year old boy. Recurrent bacterial infections. CD4 and CD8 T cells present. B cells absent, IgG, IgA, IgM absent
A. IgA deficiency
B. Common variable immunodeficiency
C. Bruton’s X linked hypogammaglobulinaemia
D. X linked hyper IgM syndrome due to CD40ligand mutation
C
•Meningococcus meningitis with family history of sibling dying of same condition aged 6
A. C9 deficiency
B. C3 deficiency with presence of a nephritic factor
C. MBL deficiency
D. C1q deficiency
Which of the following vaccinations would you NOT give to an immunosuppressed individual
- Influenza
- MMR
- Tetanus
- Pneumococcus
- HPV
MMR
Auto-immune lymphoproliferative syndrome (ALPS)
A. Single gene mutation involving MEFV and affecting the inflammasome complex, resulting in recurrent episodes of serositis
B. Mutation within the Fas pathway associated with lymphocytosis, lymphomas and auto-immune cytopenias
C. Single gene mutation involving FOXp3 resulting in abnormality of T reg cells
B
Mixed pattern auto-inflammatory / auto-immune disease with >90% heritability that results in inflammation typically involving the sacro-iliac joints and spine
A. Ankylosing spondylitis
B. APECED
C. Familial Mediterranean Fever
D. Graves Disease
E. Rheumatoid arthritis
Ank Spond
APECED
In medicine, autoimmune polyendocrine syndromes, also called polyglandular autoimmune syndrome (PGAS),[1] are a heterogeneous group[2] of rare diseases characterized by autoimmune activity against more than one endocrine organ, although non-endocrine organs can be affected.
There are three “autoimmune polyendocrine syndromes”, and a number of other diseases which have endocrine autoimmunity as one of their features.
Autoimmune polyendocrine syndrome type 1 (APECED or Whitaker’s syndrome)
Autoimmune polyendocrine syndrome type 2
The most serious but rarest form is the X-linked polyendocrinopathy, immunodeficiency and diarrhea-syndrome, also called XLAAD (X-linked autoimmunity and allergic dysregulation) or IPEX (immune dysfunction, polyendocrinopathy, and enteropathy, X-linked). This is due to mutation of the FOXP3 gene on the X chromosome.[3] Most patients develop diabetes and diarrhea as neonates and many die due to autoimmune activity against many organs. Boys are affected, while girls are carriers and might suffer mild disease.
Whitaker’s syndrome
Autoimmune polyendocrine syndrome type 1 (APECED or Whitaker’s syndrome)
Schmidt’s syndrome
Autoimmune polyendocrine syndrome type 2, a form of autoimmune polyendocrine syndrome also known as Schmidt’s syndrome,[1] or APS-II, is the most common form of the polyglandular failure syndromes.[2] It is heterogeneous and has not been linked to one gene. Rather, patients are at a higher risk when they carry a particular human leukocyte antigen genotype (HLA-DQ2, HLA-DQ8 and HLA-DR4). APS-II affects women to a greater degree than men (75% of cases occur in women).[2]
Features of this syndrome are:
Addison’s disease[3]
Primary hypothyroidism
Graves’ disease
Pernicious anaemia
Primary hypogonadism (less common)
Diabetes mellitus (type 1)
Vitiligo (less common)
Coeliac disease
Myasthenia gravis
Whitaker’s syndrome
Autoimmune polyendocrine syndrome type 1 (APS-1), also known as autoimmune polyendocrinopathy-candidiasis-ectodermal dystrophy (APECED), autoimmune polyglandular syndrome, Whitaker syndrome,[1] or candidiasis-hypoparathyroidism-Addison’s disease-syndrome,[2] is a subtype of autoimmune polyendocrine syndrome, in which multiple endocrine glands dysfunction as a result of autoimmunity. It is a genetic disorder inherited in autosomal recessive fashion due to a defect in the AIRE (Auto immune regulator) gene which is located on chromosome 21 and normally confers immune tolerance.
A 58 year old pharmacist presents with a 3 month history of skin itching associated with lethargy and loss of energy.
Physical examination is normal, but liver function tests reveal total bilirubin = 36umol/l (reference range 0-17umol/l), ALT = 28U/l (reference range 0-31U/l); Alkaline phosphatase 420U/l (reference range 30-130).
- OPTION LIST
- anti-acetyl choline receptor antibody
- anti-adrenal cortex antibody
- antibody to double stranded DNA
- anti-centromere antibody
- anti-endomysial antibody
- anti-intrinsic factor antibody
- anti-mitochondrial antibody
- anti-neutrophil cytoplasmic antibody
- anti-RNP antibody
- anti-smooth muscle antibody
●
PBC
ANti-mitochondrial
A 56 year old prison officer presents with a history of recurrent nose bleeds, haemoptysis and joint pain associated with profound lethargy.
On examination, he has crackles in his upper left lung field, and a cavitating left lung lesion is demonstrated on chest radiography.
Urine dipstick is positive for protein and blood.
- OPTION LIST
- anti-acetyl choline receptor antibody
- anti-adrenal cortex antibody
- antibody to double stranded DNA
- anti-centromere antibody
- anti-endomysial antibody
- anti-intrinsic factor antibody
- anti-mitochondrial antibody
- anti-neutrophil cytoplasmic antibody
- anti-RNP antibody
- anti-smooth muscle antibody
●
•anti-neutrophil cytoplasmic antibody
A 22 year old woman presents with joint pain and fatigue. She has an intermittent, skin-sensitive rash, and also complains of mouth ulcers. Physical examination is otherwise normal.
Urine dipstick is positive ++ protein and ++ blood.
Full blood count shows a normocytic normochromic anaemia.
- OPTION LIST
- anti-acetyl choline receptor antibody
- anti-adrenal cortex antibody
- antibody to double stranded DNA
- anti-centromere antibody
- anti-endomysial antibody
- anti-intrinsic factor antibody
- anti-mitochondrial antibody
- anti-neutrophil cytoplasmic antibody
- anti-RNP antibody
- anti-smooth muscle antibody
●
•antibody to double stranded DNA
A 30 year old plumber attends his GP complaining of feeling tired all the time. He has type I diabetes, which is currently well controlled, and a history of irritable bowel syndrome. A full blood count shows a microcytic hypochromic anaemia, and iron studies confirm iron deficiency. Vitamin D levels are within the insufficient range.
- OPTION LIST
- anti-acetyl choline receptor antibody
- anti-adrenal cortex antibody
- antibody to double stranded DNA
- anti-centromere antibody
- anti-tissue transglutaminase antibody or anti-endomyosial antibody
- anti-intrinsic factor antibody
- anti-mitochondrial antibody
- anti-neutrophil cytoplasmic antibody
- anti-RNP antibody
- anti-smooth muscle antibody
●
•anti-tissue transglutaminase antibody or anti-endomyosial antibody
A 44 year old builder presents with a history of fingers intermittently becoming very cold and white with recent development of a gangrenous tip of his finger. The skin over his fingers feels ‘tight’ and you note telangectasia on his hands.
- OPTION LIST
- anti-acetyl choline receptor antibody
- anti-adrenal cortex antibody
- antibody to double stranded DNA
- anti-centromere antibody
- anti-endomysial antibody
- anti-intrinsic factor antibody
- anti-mitochondrial antibody
- anti-neutrophil cytoplasmic antibody
- anti-RNP antibody
- anti-smooth muscle antibody
●
•anti-centromere antibody
A 19 year old student presents with a chronic, extremely itchy rash consisting of papules and vesicles which is distributed symmetrically over the extensor surfaces of her elbows, legs and buttocks. You suspect dermatitis herpetiformis.
- OPTION LIST
- anti-acetyl choline receptor antibody
- anti-adrenal cortex antibody
- antibody to double stranded DNA
- anti-centromere antibody
- anti-endomysial antibody or anti-tissue transglutaminase antibody
- anti-intrinsic factor antibody
- anti-mitochondrial antibody
- anti-neutrophil cytoplasmic antibody
- anti-RNP antibody
- anti-smooth muscle antibody
●
anti-endomysial antibody
Give 2 examples of Anti T cell monoclonal Abs
Muromonab-CD3
Basiliximab
Tocilizumab
Abatecept
What class of immune therapy is:
Muronomab-CD3
Anti T Cell monoclonal Ab
What class of immune therapy is:
Basiliximab
Anti T Cell monoclonal
What class of immune therapy is:
Tocilizumab?
Anti T Cell monoclonal
What class of immune therapy is:
Abatacept
Anti T Cell monoclonal
What class of immune therapy is:
Cyclophosphamide?
Antiproliferative
What class of immune therapy is:
Mycophenolate mofetil
Antiproliferative
What class of immune therapy is:
Azathioprine
Anti proliferative agent
What class of immune therapy is:
Tacrolimus
Inhibitor of cell signalling
What class of immune therapy is:
Ciclosporin
Inhibitor of cell signalling
What class of immune therapy is:
Sirolimus
Inhibitor of cell signalling
What class of immune therapy is:
Prednisolone
Corticosteroids
What class of immune therapy is:
Infliximab
Anti-TNF alpha monoclonal Ab
What class of immune therapy is:
Ustekinumab
Anti IL-12/23 monoclonal Ab
What is the MOA of muromonab?
Blocks CD3 on T cells
What is the MOA of Basiliximab?
Blocks CD25 (chain of IL-2R)
What is the MOA of Tocilizumab?
Blocks IL-6R
What is the MOA of Abatecept
Anti CTLA-4 Ig, blocks costimlation of T cells
What is the MOA of cyclophopshamide
Alkylatse guanine base of DNA
B>T cells
Which antiproliferative agent has a preponderence for T cells?
Mycophenolate mofetil
Which antiproliferative agents has a predeliction for B cells?
Cyclophosphamide
What is the MOA of mycophenolate mofetil
Blocks de novo nucleotide synthesis
T>B
What is the MOA of Azathioprine
Metabolised to 6-Mercaptopurine in liver.
Blocks de novo purine synthesis
What is the MOA of Tacrolimus (ciclosporin?
Inhibit calcineurin which normally activates the transcription of Il-2
Net result is a reduction in Il-2
What is the MOA of sirolimus
Blocks clonal rpolferation
What is the MOA of prednisolone
Inhibits phospholipase A2
Redcues platelet activating factor
Reduces arachidonic acid
Reduces phagocyte trafficking (hence transient increase in phagocytes)
Lymphoneia, Apoptosis of B and T cells, Ab reduction
What is the MOA of plasmapheresis
Each time 50% of patient plasma is replaced with donors
What is the MOA of Infliximab
Binds to TNF alpha
What is the MOA of Ustekinumab
Binds to p40 subunit of Il-12 and 23
Indication for Muromonab-CD3
Active rejection
Indication for basiliximab
Prevents rejection in transplantation
What is the indication for Tocilizumab
RA if anti-TNFa drugs have failed
What is the indcation for Abatecept?
RA if anti-TNF drugs have failed
What is the indcation for cylcophosphamide
Connective tissue disease (e.g. SLE)
Cancer
What is the indcation for Mycophenolate mofetil
Autoimmune disease
Vasculitis
Transplantation
What is the indcation for Azathioprine
Inflammatory and autoimmune disease
Transplantation
What is the indcation for inhibitors of Cell signalling (Tacrolimus, ciclosporin, sirolimus)
Mainly rejection prophlyaxis in transplantation
What is the indcation for corticosteroids
Used as antii-inflammatory and in autoimmune disease
What is the indcation for plasmapharesis
Goodpastures
MG etc
Ab mediated rejection
What is the indcation for infliximab
Psoriasis
Crohn’s
RA and others
What is the indcation for Ustekinumab
Psoriasis
What are the side effects of Muromonab-CD3
Fever, Leucopenia
What are the side effects of Basiliximab
GI disturbance
What are the side effects of Tocilizumab/Abatacept
Infections
Infection and cough
What are the side effects of cyclophosphamide?
Hair loss
BM suppression
Sterility
Haemorrhagic cystitis
What are the side effects of azathioprine
BM suppression (measure TPMT) Hepatotoxicity
What are the side effects of mycophenolate mofetil
Bone marrow supression
Herpes
What are the side effects of Tacrolimus
Diabetes
What are the side effects of Ciclosporin
Gingival hypertrophy
What are the side effects of corticosteroids
Diabetes
Central obesity
Adrenal suppression
Cataracts
Glaucoma
Pancreatitis
Osteoporosis
Moon Face
Acne
Hrisutism
Neutrophilia
What are the side effects of plasmapharesis
Rebound Ab production limits efficacy
What are the side effects of infliximab
TB
Lymphoma
Autoimmune phenomena
What are the side effects of Ustekinumab
Infections and cough
What is the MOA of rituximab?
Anti-CD20: reduces B cells (not plasma cells)
Indications for rituximab
Lymphoma and autoimmune disease (RA)
What is the MOA of methotrexate?
Inhibits dihydrofolate reductase reducing DNA synthesis
Indication for methotrexate
Used in autoimmune disorders e.g. RA, psoriasis, Crohn’s etc
Also used in chemotherapy and as an abortifacient
Side effects of methotrexate?
Teratogenicity
Hepatotoxicity
What is the MOA of Campath (alemtuzumab)
Monoclonal Ab that binds to CD52 found on lymphocytes resulting in depletion
Indications for alemtuzumab
CLL
Side efects of alemtuzumab?
Increased susceptibility to CMV infection
MOA of natalizumab
Anti-a4 integrin
What is a consideration for azathioprine
Some people have a TPMT polymorphism and are therefore unable to metabolise it
Which immunosuppressant can cause avascular necrosis and peptic ulceration?
Steroids
Sterility in cyclophosphamide
M>>>>>F
JC virus with mycophenolate mofetil
At risk of progressive multifocal leukoencephaloathy due to LT immunosuppression
Anti-thymocyte globulin MOA
Acts to deplete lymphocytes and modulates T cell activation
Used for allograft rejection
Features of cytokine storm
Vascular leakage:
Pulmonary oedema
Cerebral oedema
CV collapse
Poor peripheral perfusion and shcok
Etanercept MOA
TNF alpha antagonist
TNFR fusion
How to choose an immunosuppresive regime to supress T cells?
Choose one drug from each group:
Inhibitors of T cell signalling: ciclosporin, tacrolimus
Antiproliferative agent: azathioprine, MM, mehotrexate, cyclophosphamide
Blocker of cytokine production: prednisolone
What is Human Normal Ig
From pools of >1000 donors
Contains preformed IgG vs full range of organisms
Gvien every 3-4w
Indications for Human Normal Ig?
Used for priamry Ab deficiencies (CVID, Brutons etc)
Secondary deficiencies (CLL, MM, BMT)
Passive vascination
What is IVIg
Specific Ig used for post-exposure prophylaxis following exposure e.g. VZV
What are recombinant cytokines used for?
Aim to boost immune response to cancer and specific pathogens
What is the use of recombinant IFNa?
Hep C
Hep B
Kaposi’s
Hairy cell leukaemia
CML
MM
What is the use of recombinant IFNb?
Relapsing MS
What is the use of recombinant IFNg?
Chronic granulomatous disease
What is the process of allergen densitisation?
Supervised allergen administration
Reduces clinical symptoms for monoallergic disorders
Start with tiny dose and escalate until maximal dose reached
Maintenance dose given monthly for 3-5y
What are the possible side effets of immunosuppression?
Injection site reaction
Infusion reaction
INfection
Malignancy
Autoimmunity
What should be done before starting immunosuppressants?
Screen for TB, Hep B, Hep C, consider HIV infection
What is JCV?
John Cunningham Virus
Polyomavirus that can reactivate, infects and destroys oligodendrocytes causing PML
What causes malignancy in immunosuppressed?
Lymphoma- EBV
Non-melanoma skin cancers (HPV)- Kaposi’s
What are the three stages of transplant rejection
Recognition, activation, efector function
What are the 2 types of recognition in transplant rejection
Direct
Indirect
Direct recognition in transplant rejection
Donor APC presenting antigen and or MHC to recipient T cells
Acute rejection typically follows
Indirect recognition in transplant rejection
Recipient APC presenting donor Ag to recipient T cells i.e. the immune system working normally
Chronic rejection
What are the 2 forms of transplant rejection?
T cell mediated
ANtibody mediated
Rejection in mins-hrs=
Hyperacute
Rejection in weeks to mths
Acute: cellular or Ab-mediated
Rejection in months-yrs?
Chronic
Rejection in d-w with donor cells attacking host?
GvHD
Transplant rejection: mechanism in hyperacute
Preformed Ab which activates complement
Transplant rejection: mechanism in acute cellular
CD4 activating a T4 reaction
Transplant rejection: mechanism in acute antibody mediated
B cell activation- antibody attackes vessels
Transplant rejection: mechanism in chronic rejection
Immune and nonimmune mechanism
Transplant rejection: mechanism in GvHD
Donor cells attacking host
Transplant rejection, pathology in: hyperacute
Thrombosis and necrosis
Transplant rejection, pathology in acute cellular
Cellular infiltrate
Transplant rejection, pathology in acute Ab mediated
Vasculitis
C4d
Transplant rejection, pathology in chronic
Fibrosis
Transplant rejection, pathology in GvHD
Skin (rash)
Gut (D+V)
Liver (jaundice)
Transplant rejection, Mx of hyperacute
Prevention: crossmatch
Transplant rejection, Mx of acute cellular
T cell immunosuppression
Transplant rejection, Mx of antibody mediated
Ab-removal and B cell immunosuppressio
Transplant rejection, Mx of chronic
Minimise organ damage
Transplant rejection, Mx of GvHD
Prevention/immunosuppression
What are typed and matched in transplantation?
HLA-A
HLA-B
HLA-DR
Immunosuppression in transplant, drug regimen
Pretransplant: suppress T cell responses e.g. anti-CD52: Alemtuzumab or anti-CD25: basiliximab
Use immunosuppressants post-transplant to reduce infection e.g. CNI and MMF
Treat episodes of acute rejection:
Cellular- steroids
Ab-mediated: IVIG, plasma exchange, anti-C5
Process for matching ahead of trasnplant
- Determine donor and recipient blood group and HLA type
- Check recipients preformed Ab against BO and HLA (via complement dependent cytotoxicity, flow cytometry, luminex)
- Cross-match
- After transplant check for formation of new antibodies
What is the HIV receptor and how does the virus bind?
CD4
gp120
What function does the HIV protein gp41 perform?
Conformational change
What does HIV use CCR5 and CXCR4 as?
Coreceptors for viral entry
What is the innate immune response to HIV infection?
Non-specific macrophage activaiton, NK cells and complement
Stimulation of cytokines and chemokines
What is the adaptive response to HIV infection
Neutralising antibodies: anti gp120, anti-gp41
Non-neutralising Abs: antip24 gag IgG
CD8 T cells that can preent HIV entry through expressing MIP-1a, MIP1b and RANTES which block co-receptors
How does HIV damage the immune response
Remains infectious, even when Ab coated
Activated CD4 cells are killed by T cells
Activated CD4 cells are anergised
CD4 T cell memory is lost
Monocytes and dendritic cells are therefore not activated
Infected monocytes and dendritic cells are killed by virus or CTL
Quasispecies are produced due to error-prone RT, leads to immune escape
What are the stages of the HIV lifecycle
Attachment.entry
RT and DNA synthesis
Integration
Viral transcription
Viral protein synthesis
Assembly and release
Maturation
What is the natural Hx of HIV infection
Median time from infeciton to AIDS is 8-10y
Rapid progressors- 2-3y (10%)
LT nonprogressors have stable CD4 counts and nno symptoms after 10 years
Initial viral burden predicts disease progression
Dx of HIV
Screening test:
ELISA vs anti-HIV Ab
Confirmation test: detects Ab via Western Blot
Positive test requires the patient to have seroconverted (i.e. start producing Abs) - 10 weeks
Viral load monitored via PCR
CD4 count monitered via flow cytometry
Resistance testing
CD4 in AIDS
<200
HAART=
2NRTIs and PI (or NNRTI)
Atripla=
Emtricitabine, tenofovir, efavirenz
ART used in pregnancy?
Zidovudine
Antepartum PO
IV for delivery
PO to newborn for 6/52
What MOA is enfuviritide
Fusion inhibitor
What MOA is maraviroc
Attachment inhibitor
What MOA is zidovudine
NRTI
What MOA is Didanosine
NRTI
What MOA is Stavudine
NRTI
What MOA is Lamivudine
NRTI
What MOA is Zalcitabine
NRTI
What MOA is abacavir
NRTI
What MOA is emtricitabine
NRTI
What MOA is Epzicom
NRTI
What MOA is Combivir
NRTI
What MOA is Trizivir
NRTI
What MOA is Tenofovir
NRTI
What MOA is nevirapine
NNRTI
What MOA is delaviridine
NNRTI
What MOA is Efavirenz
NNRTI
What MOA is raltegravir
Integrase inhibtor
What MOA is indinavir?
PI
What MOA is nelfinavir
PI
What MOA is Riltonavir
PI
What MOA is amprenavir
PI
What MOA is fosamprenavir
PI
What MOA is Lopinavir
PI
What MOA is atazanavir
PI
What MOA is saquinavir
PI
What MOA -ines
RT inhibitors either NNRTI or NRTI
What MOA is -ravir
Integrase inhibtor
What MOA is -navir
PI
What are the side effects of enfuviritide
Local reactions: HS
What are the side effects of common NRTIs?
Generally rare:
fever
headache
GI disturbance
BMS (dzidovudine)
Peripheral neuropathy (zalcitabine, stavudine)
Mitochondrial toxicity (stavudine)
Hypersensitvity (abacavir)
What NRTI is associated with BMS
Zidovudine
Which NRTIs are associated with peripheral neuropathy
Zalcitabine, stavudine
Which NRTIs are associated with mitochondrial toxicity
Stavudine
What are the side effects of tenofovir?
Bone and renal toxicity
What are the side effects of nevirpaine
Hepatitis and rash
What are the side effects of delaviridine
Rash
What are the side effects of efavirenz
CNS effects
What are the side effects of the PIs
Hyperlipidaemias
Fat redistribution
T2DM
A 26 year old male who has been suffering from ‘flu-like’ symptoms with fever presents to the GP after developing skin rash in the last few days.
A.
IgE mediated anaphylaxis
B.
Chronic urticaria
C.
Allergic asthma
D.
Urticarial vasculitis
E.
Mast cell degranulation
F.
Panic attack
G.
C1 inhibitor deficiency
H.
Extrinsic allergic alveolitis
I.
Coeliac disease
J.
Idiopathic angioedema
K.
Acute urticaria
Acute urticaria
A 55 year old man with history of angina was advised to take a tablet before a long flight. After taking the pill, he suddenly finds that he has difficulty breathing, feels nauseous and is itching.
A.
IgE mediated anaphylaxis
B.
Chronic urticaria
C.
Allergic asthma
D.
Urticarial vasculitis
E.
Mast cell degranulation
F.
Panic attack
G.
C1 inhibitor deficiency
H.
Extrinsic allergic alveolitis
I.
Coeliac disease
J.
Idiopathic angioedema
K.
Acute urticaria
Q: Actually, just realised that he probably took aspirin which causes a non- IgE mediated anaphylaxis i.e. mast cell degranulation A: Romesh Yes, it’s to do with aspirin. Although i have seen doctors advising patients to take aspirin before flight to prevent DVT, it is not generally helpful as aspirin is thought to be involved in preventing ARTERIAL thrombosis, because it is an anti platelet, and not venous thrombosis. Heparin would be the better option for venous thrombosis. A: Mark I agree. just to add ARTERIAL thromboses: “white” because platelets aggregate first.. this causes stasis which triggers fibrin formation (coagulation cascade) VENOUS thromboses: red because they are more fibrin rich than platelet rich.
A 40 year old man complains of loss of smell with nasal itching and discharge over 4 weeks. He also describes morning sneezing. He is otherwise in good health. On examination his nasal mucosa are swollen and hyperaemic.
A.
Intraarticular corticosteroids
B.
Inhaled corticosteroids
C.
IM adrenaline 0.5 mL of 1:1000
D.
IM adrenaline 1mL of 1:1000
E.
PO antihistamines
F.
IM adrenaline 1mL of 1:10000
G.
Venom immunotherapy
H.
Intranasal antihistamines
I.
Intracardiac adrenaline
J.
IV adrenaline 0.3mL of 1:1000
K.
IV antihistamines
L.
None of the above
M.
Inhaled antihistamines
Oral antihistamines and intranasal corticosteroids are the mainstay of treatment of mild allergic rhinitis
(As intranasal corticosteroid is not an option available, the “single best” answer here is oral antihistamines.)
A 35 year old woman presents with a two day history of a red itchy skin rash which started soon after her first scuba-diving lesson. She is otherwise well
- The most important treatment of anaphylaxis is adrenaline, which should be given intramuscularly. (Note for final year pharm: 1:1000 means 1mg/mL; 1:10000 means 0.1mg/mL ; 1% means 1g/dL) 2. Severe acute urticaria is effectively treated with a short course of oral anti-histamines 3. Contact hypersensitivity should be treated by avoidance of the sensitising agent, in this case nickel 4. Oral antihistamines and intranasal corticosteroids are the mainstay of treatment of mild allergic rhinitis. (As intranasal corticosteroid is not an option available, the “single best” answer here is oral antihistamines.) 5. Intramuscular adrenalin should be used in patients with severe local angioedema with secondary acute respiratory tract obstruction. However this is not always effective in ACE inhibitor-induced angioedema, and some patients will require intubation. Always stop the causative agent!
Acts on hepatocytes to induce synthesis of acute phase proteins in response to bacterial infection
IL6
Goodpasture’s syndrome
A.
Type IV – Complement mediated
B.
Type III – Immune complex mediated
C.
Type III – complement mediated
D.
Type II – Antigen mediated
E.
Not an autoimmune disease
F.
Type IV – T-cell mediated
G.
Type II – Antibody mediated
H.
Type III – T-cell mediated
Type II Ab mediated
Severe Combined Immunodeficiency
A.
IFN Receptor 1 gene
B.
CD40 Ligand gene
C.
IL12 gene
D.
Chromosome 22q11
E.
Bruton’s tyrosine kinase (Btk) gene
F.
CD3 mutation
G.
WASP gene
H.
MHC Class II
I.
IL-2 receptor
I.
IL-2 receptor
Wiskott-Aldrich Syndrome
A.
IFN Receptor 1 gene
B.
CD40 Ligand gene
C.
IL12 gene
D.
Chromosome 22q11
E.
Bruton’s tyrosine kinase (Btk) gene
F.
CD3 mutation
G.
WASP gene
H.
MHC Class II
I.
IL-2 receptor
WASp gene
Hyper IgM sydrome
A.
MHC Class III
B.
Bruton’s tyrosine kinase (Btk) gene
C.
CD3 mutation
D.
IFN Receptor 1 gene
E.
IL12 gene
F.
Chromosome 22q11
G.
Adenosine Deaminase (ADA) gene
H.
IL-2 receptor
I.
CD40 Ligand gene
J.
WASP gene
CD40 Ligand gene
In acute rejection, these are produced as a result of the activation of neutrophils and macrophages
A.
High dose corticosteroids
B.
Amino acids
C.
Granzyme B
D.
Hypotension
E.
HLA DR > A > B
F.
CD17+ T cells
G.
HLA A > B > DR
H.
HLA type
I.
ABO blood type
J.
CD8+ T cells
K.
Diuretics
L.
HLA DR > B > A
M.
Free radicals
N.
Diabetes
O.
CD4+ T cells
P.
Antibiotics
Q.
Interferon gamma
R.
IV Immunoglobulins and Plasmapheresis
S.
Hypertension
T.
Hyperacute
Free radicals
Risk factor for chronic allograft rejection
A.
High dose corticosteroids
B.
Amino acids
C.
Granzyme B
D.
Hypotension
E.
HLA DR > A > B
F.
CD17+ T cells
G.
HLA A > B > DR
H.
HLA type
I.
ABO blood type
J.
CD8+ T cells
K.
Diuretics
L.
HLA DR > B > A
M.
Free radicals
N.
Diabetes
O.
CD4+ T cells
P.
Antibiotics
Q.
Interferon gamma
R.
IV Immunoglobulins and Plasmapheresis
S.
Hypertension
T.
Hyperacute
Hypertension
The 3 most important HLA types to screen for in renal transplantation when matching donor and recipient, in order of importance
A.
High dose corticosteroids
B.
Amino acids
C.
Granzyme B
D.
Hypotension
E.
HLA DR > A > B
F.
CD17+ T cells
G.
HLA A > B > DR
H.
HLA type
I.
ABO blood type
J.
CD8+ T cells
K.
Diuretics
L.
HLA DR > B > A
M.
Free radicals
N.
Diabetes
O.
CD4+ T cells
P.
Antibiotics
Q.
Interferon gamma
R.
IV Immunoglobulins and Plasmapheresis
S.
Hypertension
T.
Hyperacute
HLA DR > B > A
Example of a vaccine that should NOT be given to a severely immunocompromised patient.
A.
Infliximab
B.
Diptheria, Tetanus, Pertussis vaccine
C.
Tacrolimus
D.
Bee/wasp venom allergy
E.
Blocking cytokine synthesis
F.
Polio vaccine
G.
Plasmapheresis
H.
Mycophenolate mofetil
I.
Influenza type B vaccine
J.
Inhibition of DNA synthesis
K.
Bone marrow suppression
L.
Atopic dermatitis
M.
Goodpasture’s syndrome
Polio
Prevents DNA replication especially of T cells
A.
Chloramphenicol
B.
Cyproterone acetate
C.
Dobutamine
D.
Immunoglobulins
E.
Perindopril
F.
Thyroxine
G.
Mycophenolate mofetil
H.
Ribavirin
I.
Metolazone
J.
Cyclophosamide
K.
Infliximab
L.
Prednisolone
M.
Gentamicin
N.
Ciclosporin
Mycophenolate mofetil
Methotrexate
A.
Bone marrow depression
B.
Anorexia
C.
Anaphylaxis
D.
Lethargy
E.
Pneumonitis, pulmonary fibrosis and cirrhosis
F.
Hair loss
G.
Dysrhythmias
H.
Ototoxicity
I.
Hypertension and reduced GFR
J.
Hypertension
Pneumonitis, pulmonary fibrosis and cirrhosis
For methotrexate (MTX) induced cirrhosis monitor serum procollagen III rather than doing liver biopsy. MTX is given once WEEKLY as maintenance therapy in autoimmune disease; more often and you’re looking at anti-tumour regimens. Remember to replace folate.
A 5-month-old boy is referred to a paediatrician after suffering with recurrent
infections since his birth. His mother has noticed increased irritability. Blood
tests reveal a neutrophil count of 350/μL. NBT test is normal.
A Kostmann syndrome
B Severe combined immunodeficiency
C Hyper IgM syndrome
D Leukocyte adhesion deficiency
E Protein-losing enteropathy
F Cyclic neutropenia
G Bruton’s agammaglobulinaemia
H Di George’s syndrome
I AIDS
Kostmann syndrome (severe congenital neutropenia; A) is a congenital
neutropenia as a result of failure of neutrophil maturation. This results
in a very low neutrophil count (less than 500/μL indicates severe neutropenia)
and no pus formation. Kostmann syndrome is usually detected
soon after birth. Presenting features may be non-specific in infants,
including fever, irritability and infection. The nitro-blue-tetrazolium
(NBT) test can help with diagnosis; the liquid turns blue due to the normal
presence of NADPH. In Kostmann syndrome, NBT test is positive
and therefore normal.
CATCH
Features of Di George’s
Cardiac abnormalities
Atresia
Thymic aplasia
Cleft palate
Hypocalacaemia
A 4-year-old boy is referred to a paediatrician after suffering recurrent chest
infections over the preceding few months. The boy has a history of eczema as
well as recurrent nose bleeds. Blood tests reveal a reduced IgM level but raised
IgA and IgE levels.
A Selective IgA deficiency disease
B Common variable immunodeficiency
C Nephrotic syndrome
D Bare lymphocyte syndrome
deficiency
E Sickle cell anaemia
F Chronic granulomatous
G Reticular dysgenesis
H Wiskott–Aldrich syndrome
I Interferon-gamma receptor
Wiskott–Aldrich syndrome (WAS; H) is an X-linked condition which is
caused by a mutation in the WASp gene; the WAS protein is expressed
in developing haematopoietic stem cells. WAS is linked to the development
of lymphomas, thrombocytopenia and eczema. Clinical features
include easy bruising, nose bleeds and gastrointestinal bleeds secondary
to thrombocytopenia. Recurrent bacterial infections also result. Blood
tests reveal a reduced IgM level and raised IgA and IgE levels. IgG levels
may be normal, reduced or elevated.
Blood
tests reveal a reduced IgM level and raised IgA and IgE levels. IgG levels
may be normal, reduced or elevated.
Wiskott Aldrich Syndrome
Blood tests reveal
a reduced B-cell count, a normal/reduced IgM level and decreased levels
of IgA, IgG and IgE.
A
mutation of MHC III causes aberrant class switching, increasing the risk
of lymphoma and granulomas.
CVID
A 45-year-old man undergoes a heart transplant due to end-stage heart failure.
Seventy-two hours after the operation, the patient shows signs of organ rejection
which is resistant to corticosteroid therapy. A mouse monoclonal antibody
is administered to save the transplant.
B Corticosteroids
C Cyclosporine A
D Azathioprine
E Sirolimus
F OKT3
G IL-2 receptor antibody
H Tacrolimus
I Anti-lymphocyte antibody
OKT3 (muromonab-CD3; F) is a mouse monoclonal antibody targeted
at the human CD3 molecule used to treat rejection episodes in patients
who have undergone allograft transplantation. Administration of the
antibody efficiently clears T cells from the recipient’s circulation, T cells
being the major mediator of acute organ rejection. Primary indications
include the acute corticosteroid-resistant rejection of renal, heart
and liver transplants. Anaphylaxis can result given a murine protein
is introduced to the recipient. OKT3 can also bind to CD3 on T cells,
stimulating the release of TNF-α and IFN-γ causing cytokine release
syndrome, which if severe, can be fatal.
A 32-year-old woman undergoes a bone marrow transplant for chronic lymphoblastic
leukaemia. She is prescribed a medication that inhibits calcineurin. On
examination, the patient has gum hyperplasia
A HLA-matching
B Corticosteroids
C Cyclosporine A
D Azathioprine
E Sirolimus
F OKT3
G IL-2 receptor antibody
H Tacrolimus
I Anti-lymphocyte antibody
Cyclosporine A (C) is an important immunosuppressive agent in the organ
transplant arena, which inhibits the protein phosphatase calcineurin. This in
turn inhibits IL-2 secretion from T cells, a cytokine which stimulates T cell
proliferation. Another proposed mechanism of action involves the stimulation
of TGF-β production. TGF-β is a growth-inhibitory cytokine, the
production of T cells is reduced, hence minimizing organ rejection. Adverse
effects include nephrotoxicity, hepatotoxicity, diarrhoea and pancreatitis.
On examination, patients taking cyclosporine A may have gum hyperplasia.
E) inhibits T-cell proliferation by binding to
FK-binding protein-1A (FKBP-1A). Its advantage lies in its low nephrotoxicity
in comparison to other immunosuppressive agents.
Sirolimus
calcineurin inhibitor that inhibits T-cell proliferation
by binding to FKBP-1A.
In contrast to sirolimus, which affects
T-lymphocyte clonal proliferation, tacrolimus targets T-cell activation.
IL-2 receptor antibody (targets the CD25 of IL-2 receptors
expressed on the surface of activated T cells. It is especially used in
kidney transplant patients to prevent organ rejection.
Daclizumab
pulmonary
fibrosis, pericardial effusion, rheumatoid nodules and splenomegaly
Felty’s syndrome
A 52-year-old man is referred to a gastroenterologist with itchy skin and
malaise. On examination, the man has bruising on his arms and legs.
A Anti-mitochondrial
B c-ANCA
C Anti-cardiolipin
D Anti-ribonucleoprotein
E Anti-glutamic acid decarboxylase
F Anti-Ro
G Anti-nuclear
H Anti-intrinsic factor
I Anti-endomysial
Anti-mitochondrial (A) antibodies are associated with primary biliary
cirrhosis (PBC), and are immunoglobulins against mitochondria in cells
of the liver. PBC is an autoimmune disease of unknown cause characterized
by lymphocytic destruction of the bile canaliculi of the liver;
build-up of bile leads to fibrosis and eventually cirrhosis. Clinical features
include pruritis (increased bile acids in circulation) as well as the
effects of reduced absorption of fat soluble vitamins (vitamin D, osteomalacia;
vitamin K, bruising; vitamin A, blindness).
primarily affects the nose (saddle-nose deformity due to perforated
septum; epistaxis), lungs (pulmonary haemorrhage) and kidneys (glomerulonephritis).
Wegener’s granulamatosis, a vasculitic disease
that is in severe cases life threatening. c-ANCA is directed towards proteinase
3 (PR3) within the neutrophil cytoplasm.
a medium and small-vessel autoimmune
vasculitis. Blood vessels of the lungs, gastrointestinal system
and peripheral nerves are most commonly affected
p-ANCA (perinuclear anti-neutrophil cytoplasmic antibodies; B) is a
feature of Churg–Strauss syndrome
A 34-year-old man who has been taking amoxicillin for pneumonia has developed
tiredness and palpitations since taking the medication. Blood tests reveal a
normocytic anaemia and direct antiglobulin test is positive.
A Stony fruit
B HBsAg
C Myelin basic protein
D Rhesus antigens
E Glycoprotein IIb–IIIa
F Peanuts
G Antiserum
H Synovial membrane antigens
I Poison ivy
Rhesus antigens (D) are found on the surface of erythrocytes. The rhesus
(Rh) blood group system is clinically the most important after the ABO
system; the most commonly used Rh antigen is the D antigen, signifying
whether a patient is Rh positive or negative. Antibodies directed
against the Rh antigen results in autoimmune haemolytic anaemia
(AIHA; type II hypersensitivity reaction). Most commonly the cause is
idiopathic, however, chronic lymphocytic leukaemia, systemic lupus
erythematosus and drugs (methyldopa and penicillin) can trigger AIHA.
Direct antiglobulin test is positive.
vasculitis of small and medium sized vessels. Immune
complexes (type III hypersensitivity reaction) are deposited within such
vessels leading to fibrinoid necrosis and neutrophil infiltration; as a
result the vessel walls weaken and there is aneurysm development.
Investigations will reveal a raised ESR, CRP and immunoglobulin level.
pANCA is also associated
Angiogram will reveal multiple
aneurysms. Corticosteroids and cytotoxic agents are required to control
disease progression.
HBsAg (B) may be associated with the development of polyarteritis
nodosa (PAN),
Cyclophosphamide and bladder
Associated with TCC
A 56-year-old man who is undergoing kidney transplant surgery is given medication
to prevent allograft rejection. The drug prevents guanine synthesis to
induce immunosuppression.
A Cyclophosphamide
B Mycophenolate mofetil
C Basiliximab
D Abatacept
E Rituximab
F Efalizumab
G Infliximab
H Ustekinumab
I Denosumab
For
Mycophenolate mofetil (B) is the prodrug of mycophenolic acid which
inhibits inosine monophosphate dehydrogenase (IMPDH), an enzyme
required in guanine synthesis; impaired guanine synthesis reduces the
proliferation of both T and B cells, but T cells are affected to a greater
extent. Mycophenolate mofetil is indicated as an immunosuppressive
agent in transplant patients as well as an alternative to cyclophosphamide
in the treatment of autoimmune diseases and vasculitides. Side
effects include bone marrow suppression (particularly low white blood
cells and platelets) as well as herpes virus reactivation.
A 56 year old with known systemic lupus erythematosus has been treated with
long-term steroids. The patient presents to a rheumatologist with back pain and
a DEXA scan confirms osteoporosis.
A Cyclophosphamide
B Mycophenolate mofetil
C Basiliximab
D Abatacept
E Rituximab
F Efalizumab
G Infliximab
H Ustekinumab
I Denosumab
Denosumab (I) is an antibody directed towards the RANK ligand in
bones. Osteoblasts are responsible for bone formation, whilst osteoclasts
(which contain the cell surface receptor RANK) break down
bone. Inhibition of RANK by denosumab therefore inhibits osteoclast function
and differentiation, thereby preventing the breakdown of bone.
Denosumab is indicated in the treatment of osteoporosis but is also used
in the management of multiple myeloma and bone metastases. Toxicity
can predispose to respiratory and urinary tract infections.
an antibody against CD11a on T cells; it inhibits the
migration of T cells. It is indicated in the treatment of psoriasis.
Efalizumab
is an antibody directed towards IL-2α receptor (CD25)
which causes reduction in T-cell proliferation. It is used as a prophylactic
treatment of allograft rejection.
Basiliximab (C)
an antibody to the p40 subunit of IL-12 and IL-23
thereby preventing T-cell and natural-killer cell activation. It is used in
the treatment of psoriatic arthritis
Ustekinumab (H
Characteristically there is mesangial
proliferation with deposition of IgA together with alternative pathway
factors C3 and properdin.
IgA nephropathy (Berger’s disease; E) is the most common cause of
glomerunephritis in the developed world. The condition occurs after a
gastrointestinal or upper respiratory infection; potential offenders are
postulated to include Haemophilus influenzae, hepatitis B virus and
cytomegalovirus.
glomerulonephritis include diffuse hypercellularity and diffuse swelling
of the mesangium and glomerular capillaries. Influx of neutrophils
and macrophages may reveal crescent formation on histology. Direct
immunofluorescence
reveals the sub-epithelial deposition of IgG and C3.
Post-streptococcal glomerulonephritis (H) is usually caused by a preceding
group A β haemolytic streptococcus pharyngitis.
characterized by the deposition of IgG, IgM, IgA
and C3 in the sub-endothelial segment of the glomerular basement
membrane and in the mesangium.
Lupus nephritis
defined by mesangial
cell proliferation with thickening of the capillaries. Two types exist:
type 1 in which there is classical and alternative complement pathway
activation and type 2 that is associated with only alternative pathway
activation.
Membranoproliferative glomerulonephritis (F)
the gold standard for investigating such type I
hypersensitivity reactions.
Skin prick test
The test involves a few drops of purified
allergen being pricked onto the skin. Allergens which are tested for
include foods, dust mites, pollen and dust. A positive test is indicated by
wheal formation, caused by cross-linking of IgE on the mast cell surface
leading to histamine release.
A 12-year-old girl is referred to a paediatrician after suffering with allergies to
a number of foods including peanuts and eggs. Her mother wants to check if
she is allergic to any other foods, inhalants or specific materials, so that she can
be prevented from coming into contact with potential allergens.
A Histocompatibility testing
B Immunofluorescence
C Latex fixation test
D Radioallergosorbent test
E Patch testing
F Kveim test
G Skin prick test
H Western blot
I Direct antiglobulin test
Skin prick test (G) is the gold standard for investigating such type I
hypersensitivity reactions. The test involves a few drops of purified
allergen being pricked onto the skin. Allergens which are tested for
include foods, dust mites, pollen and dust. A positive test is indicated by
wheal formation, caused by cross-linking of IgE on the mast cell surface
leading to histamine release.
A 5-year-old boy presents to accident and emergency with purpura on his legs
and buttocks, joint pain and abdominal pain. The boy’s mother states that the
child had suffered from a sore throat approximately 1 week previously. The doctor
would like to perform an investigation to make sure of the diagnosis.
A Histocompatibility testing
B Immunofluorescence
C Latex fixation test
D Radioallergosorbent test
E Patch testing
F Kveim test
G Skin prick test
H Western blot
I Direct antiglobulin test
Immunofluorescence (B) is an immunological technique used in conjunction
with fluorescence microscope. Fluorophores (fluorescent
chemical compounds) attached to specific antibodies are directed at
antigens found within a biological specimen, most commonly a biopsy
sample, to visualize patterns of staining. For example, in Henoch–
Schönlein purpura, anti-IgA antibody will demonstrate IgA deposits in
the capillary walls of the specimen. Immunofluorescence may be direct
(use of a single antibody bound to a single fluorophore) or indirect
(secondary antibody carrying the fluorophore binds to the primary
antibody).
an agglutination technique used in the detection
of antibodies. It is used in the detection of rheumatoid factor.
Latex fixation test
radioimmunoassay test for a variety
of potential allergens. The test involves the use of radio-labelled antihuman
IgE; the antibody is added, which attaches to the IgE bound to
the insoluble allergen.
Radioallergosorbent test
Patch testing
useful test to determine the causative allergen in
contact dermatitis.
A 62-year-old woman sees her GP for a regular check-up. On examination, she
has notable deformities of her hands, including swan-neck and Boutonniere
deformities of her fingers. Blood tests reveal a raised CRP. Which of the following
investigation results will most likely feature?
A Reduced AH50 and normal CH50
B Reduced C1 inhibitor
C Reduced C3 and C4
D Reduced C3 and normal C4
E High CH50
The complement system is composed of the classical, lectin and alternative
pathways. These individual pathways culminate in the formation of
the membrane attack complex (MAC), which traverses cell surface membranes
of pathogens, causing cell lysis. Components of the complement
system can be quantified in order to differentiate possible diagnoses.
CH50 (total complement activity) measures the level of factors of the
classical and final pathways (C1–C9). As complement factors are acute
phase proteins, a high CH50 (E) indicates acute or chronic inflammation.
Together with the raised CRP and clinical features, this patient is likely
to suffer from rheumatoid arthritis.
anti-nephritic antibodies cause consumption of complement factors,
especially C3. As a result, complement profiling reveals a reduced C3
but normal C4
membranoproliferative glomerulonephritis (MPGN),
involves factors C3, B, D and P.
AH50
A 23-year-old man presents to his GP with recent onset diarrhoea, fatigue and
weight loss. The patient suggests that his symptoms are worsened after eating
bread or rice. Which human leukocyte antigen is most likely to be associated
with his disease process?
A HLA B27
B HLA DR2
C HLA DR3
D HLA DR4
E HLA DQ2
HLA DQ2 (E) represents a risk factor for coeliac disease (HLA DQ8 is also a risk factor but to a lesser extent).
A 3-year-old Afro-Caribbean boy is referred to a paediatrician after concerns
about his recurrent chest infections. The child’s hair slowly fell out and there is
evidence of depigmentation of his skin. Blood tests reveal hypocalcaemia and
high TSH levels. Which component of the immune tolerance system is likely to
be dysfunctional?
A Regulatory T cell
B TGF-β
C Autoimmune regulator
D Dendritic cells
E IL-10
The autoimmune regulator (AIRE; C) is also present
within the thymus and presents T-cell receptors with a range of
organ-specific antigens. If T-cell receptors bind to such antigens, they
swiftly die via apoptosis. Autoimmune polyendocrine syndrome type 1
(APECED; associated with mild immune deficiency, dysfunctional parathyroid
gland/adrenal gland, hypothyroidism, gonadal failure, alopecia
and vitiligo) results from mutations in the AIRE gene. The child in this
scenario has features of APECED
immunodysregulation polyendocrinopathy enteropathy
X-linked syndrome.
The mechanisms of central tolerance are, however, not fail-safe, and
so peripheral systems exist to remove potential auto-reactive T cells.
Regulatory T cells (A) mature in the thymus and are those that express
CD4, CD25 and Foxp3 on the cell surface. Abnormal Foxp3 leads to the
development of immunodysregulation polyendocrinopathy enteropathy
X-linked syndrome.
Defective immunoregulation=
Aberrant function of regulatory T cells which bear CD4., CD25 and Foxp3 and are reponsible for maintaining peripheral tolerance
What is T Cell bypass
T-cell bypass (B) involves the generation of a novel autoantigen epitope.
Autoantigens are physiologically internalized by B cells, which are in
turn presented to T-helper cells; the B cell is suppressed from producing
autoantibodies. If the complex autoantigen is modified, a new epitope is
provided for T cells to stimulate antibody production by B cells. Triggers
to this modification include drugs and infection, such as Mycoplasma
pneumoniae inducing autoimmune haemolytic anaemia by modifying
erythrocyte surface proteins.
Mecahnism of Dressler’s syndrome
Release of ‘hidden’ self antigens (D) may
occur after damage to an organ and causes release of intracellular proteins
which have never been exposed to the immune system. This is the
case post-myocardial infarction, where release of proteins leads to the
generation of autoantibodies against cardiac myocytes (Dressler’s syndrome),
causing pericarditis
A 29-year-old woman presents to her GP with recent onset joint pain and tiredness.
On examination she has a malar rash. Further blood tests reveal she is antinuclear
antibody and anti-double stranded DNA positive. Which component of
the complement system is she most likely to be deficient in?
A C3
B C4
C C6
D C9
E C1 inhibitor
This patient demonstrates symptoms, signs and diagnostic features consistent
with systemic lupus erythematosus (SLE) and is therefore most
likely to have a deficiency of the classical pathway such as C4 deficiency
(B). Other possible deficiencies in this pathway include C1q, C1r and
C1s and C2. The classical pathway is responsible for clearing immune
complexes and apoptotic cells; patients who have deficiencies in this
pathway therefore have a greater risk of developing immune complex
disease such as SLE.
C3 deficiency
C3 (A) is a common factor in both the classical
and alternative pathways. Deficiency of C3 leads to recurrent pyogenic
infections as there is no C3b (produced via C3 convertase) available to
opsinize bacteria. C3 deficiency also leads to decreased C3a production,
an anaphylatoxin that mediates inflammation.
C9 deficiency vs deficiency in other components of the MAC
While C9 (D) also forms part of the MAC, patients deficient in C9 still retain some ability to clear encapsulated bacterial infection, albeit at a slower rate. Therefore, patients deficient in C9 are usually asymptomatic
A 24-year-old man with a history of coeliac disease visits his GP after several
bouts of chest and gastrointestinal infections in the past few years. Although
the infections are mild, the patient is worried about the cause. What is the
diagnosis?
A Severe combined immunodeficiency
B Bruton’s agammaglobulinaemia
C Hyper IgM syndrome
D Selective IgA deficiency
E Common variable immunodeficiency
IgA specifically provides mucosal immunity, primarily to the respiratory
and gastrointestinal systems. Selective IgA deficiency (D) results from a
genetic inability to produce IgA and is characterized by recurrent mild
respiratory and gastrointestinal infections. Patients with selective IgA
deficiency are also at risk of anaphylaxis to blood transfusions due to
the presence of donor IgA. This occurs especially after a second transfusion;
antibodies having been created against IgA during the primary
transfusion. Selective IgA deficiency is also linked to autoimmune diseases
such as rheumatoid arthritis, systemic lupus erythematosus and
coeliac disease.
A mutation of MHC III causes aberrant
class switching, increasing the risk of lymphoma and granulomas.
Clinical features include bronchiectasis and sinusitis. Blood tests reveal
a normal IgM level but decreased levels of IgA, IgG and IgE.
Common variable immunodeficiency
CD vs UC and enteropathy associated immune deficiency
CD more commonly involved as small bowel is site of protein absorption
Why is prematuriy a cause of secondary immunodeficiency?
Prematurity (E) is a cause of secondary immunodeficiency
as IgG is transferred across the placenta during the final
2 months of pregnancy. Premature babies will have had less IgG transferred
as a fetus. As a result, such babies will be at greater risk of infection
before their own immune systems begin to mature (
approximately
4 months after birth).
A 12-year-old girl has developed a runny nose, itchy eyes and nasal congestion
during the summer months for the past 4 years. She is prescribed anti-histamines
to help her symptoms. Which of the following cells is responsible for the initial
encounter with the allergen?
A Mast cell
B B cell
C Macrophage
D TH1 cell
E TH2 cell
Type I hypersensitivity reactions are mediated by IgE and are associated
with allergy and anaphylaxis. The mechanism behind the development
of type I hypersensitivity reactions begins with the presentation of the
allergen to professional antigen presenting cells. Professional antigen
presenting cells include macrophages (C), dendritic cells and B cells. For
example, if an allergen is taken up by a macrophage, it is processed
intracellularly and peptides are presented via major histocompatibility
complex on the cell surface to T cells of the TH2-cell (E) subclass
A 14-year-old girl with a history of eczema presents to accident and emergency
with itching and tingling of her lips and tongue. The girl’s lips are evidently
swollen. All observations are normal. The doctor believes her condition is due to
cross-reactivity of allergens. What is the most likely trigger for her allergy?
A Penicillin
B Eggs
C Nickel
D Dust mite
E Fruit
This patient has signs and symptoms confined to her mouth. Together
with the doctor’s suspicions regarding the underlying pathogenesis, oral
allergy syndrome (OAS) is the most likely diagnosis. OAS occurs secondary
to cross-reactivity of antigens inhaled in the mouth, otherwise
known as pollen–food allergy. For example, a patient may be sensitized
to birch pollen; when pollen is breathed in, IgE is created which
cross-reacts with fruit (E) which has been ingested causing release of
histamine from mast cells resulting in local inflammation
What is significant re treatment of T1 vs T4 HS
Histamines not involved in T4 so antihistamines don’t work
Double-blind challenges (C) are reserved for
food allergies
where there is some doubt after a skin prick or RAST test. This must be
conducted at a centre where necessary equipment is available in case of
anaphylaxis.
The presence
of anti-Smith antibodies suggests
interstitial lung disease involvement.
A 34-year-old woman notices an itchy and desquamating, erythematous rash on
her wrist, which has emerged approximately 3 days after wearing a new bracelet.
Which cytokine is the first to be released during the initial exposure to the
allergen?
A IL-10
B IFN-γ
C IL-2
D TNF-α
E IL-12
Type IV hypersensitivity (delayed type) reactions are those that are
mediated by T cells of the immune system. These types of reactions
require two exposures to the allergen. During the first encounter,
antigen presenting cells such as macrophages engulf the allergen and
presents peptides on the cell surface via major histocompatibility complex.
CD4+ T cells recognize the peptide and bind to the macrophage.
The macrophage then releases IL-12
During the second exposure,
the macrophage will once again take up the allergen and present
peptide to CD4+ T cells. On this occasion however, the sensitized
memory T cell releases IFN-γ (B), IL-2 (C) and IL-3 thereby activating
macrophages, inducing the production of TNF-α (D); the result is tissue
injury and chronic inflammation.
A 56-year-old woman presents to her GP with blurry vision. On examination the
woman has some bilateral weakness in her legs. The patient mentions that her
vision seems to become more blurry just after she has had a bath. What is the
most likely target in this disease process?
A Pancreatic β-cell proteins
B Nickel
C Proteolipid protein
D Synovial membrane proteins
E Tuberculin
Type IV hypersensitivity reactions are mediated by T cells and have
a delayed onset. Proteolipid protein (C) and myelin basic protein are
oligodendrocyte proteins implicated in the pathogenesis of multiple
sclerosis (MS). Multiple sclerosis is a demyelinating disease in which the
myelin sheaths surrounding neurons of the brain and spinal cord are
destroyed. Associated with the disease process is the antigenic stimulation
of CD4+ T cells which in turn activate CD8+ cytotoxic T cells and
macrophages; these are directed at oligodendrocyte proteins, causing
destruction of oligodendrocytes and myelin. Clinical features of MS
include optic neuritis, urinary/bowel incontinence, weakness of the
arms/legs and dysphagia. Uhthoff’s phenomenon describes the worsening
of symptoms that occurs after exposure to higher than ambient
temperatures.
Uhthoff’s phenomenon
Worsening of MS symptoms after exposure to higher than ambient temperature
A 40-year-old diabetic man is to undergo a kidney transplant as a consequence
of stage 5 chronic kidney disease. The patient has an identical twin who is willing
to donate a kidney, and has been HLA matched at all loci. Which term best
describes the type of organ transplant proposed?
A Autograft
B Split transplant
C Allograft
D Isograft
E Xenograft
transplant from the patient’s twin is known
as an isograft (D); as the two individuals will have a similar genetic
profile and the organ has been matched for human leukocyte antigen
(HLA), chance of rejection is rare.
A 56-year-old woman presents to her GP with blurry vision. On examination the
woman has some bilateral weakness in her legs. The patient mentions that her
vision seems to become more blurry just after she has had a bath. What is the
most likely target in this disease process?
A Pancreatic β-cell proteins
B Nickel
C Proteolipid protein
D Synovial membrane proteins
E Tuberculin
C Proteolipid protein
Prednisolone vs methylprednisolone in transplant rejection
Prednisolone used prophylacitcally
Methylprednisolone used to treat
How are macrophages implicated in HIV CNS infection
Macrophages infected by HIV are not destroyed but are used as replicating
reservoirs as well as a means of gaining entry to the central nervous
system as macrophages are able to cross the blood–brain barrier
A 3-year-old boy is referred to a paediatrician after experiencing recurrent chest
infections. Blood tests demonstrate a reduced B-cell count as well as low IgA,
IgM and IgG levels. Genetic testing reveals a defect in the BTK gene. What is the
best therapeutic modality for this child?
A IFN-α
B IFN-β
C IFN-γ
D Intravenous IgG
E Haematopoietic stem cell transplant
Intravenous IgG (D) is not a cure for Bruton’s agammaglobulinaemia but prolongs survival. Treatment must be continued throughout life. Intravenous IgG is also used in the treatment of hyper IgM syndrome, common variable immunodeficiency as well as secondary antibody deficiencies.
A 49-year-old woman with known rheumatoid arthritis is seen in the rheumatology
clinic. She has been taking a medication over a long period of
time which is used to control proliferation of her white blood cells. The
patient explains that she has been feeling tired recently and has suffered
with low moods. Routine blood tests reveal she has a macrocytic megaloblastic
anaemia
A Cyclophosphamide
B Mycophenolate mofetil
C Azathioprine
D Methotrexate
E Cisplatin
methotrexate (D) as the correct answer.
Methotrexate is an anti-metabolite and anti-folate drug indicated for
the treatment of cancer as well as autoimmune diseases including rheumatoid
arthritis and systemic lupus erythematosus. Methotrexate inhibits
dihydrofolate reductase (DHFR), an enzyme involved in the synthesis
of the nucleoside thymidine; thymidine is essential for DNA synthesis.
As folate is required for the synthesis of purine, production of this base
is also disrupted. Ultimately, proliferation of leukocytes is interrupted.
Side effects include those of folate deficiency (macrocytic megaloblastic
anaemia, loss of appetite, tiredness, weakness and depression). The
low white cell count that results predisposes to infection; this is an
adverse effect of all anti-proliferative drugs
replication; cyclophosphamide affects B-cell replication more than
T cells. Complications of therapy include bone marrow suppression,
hair loss and carcinogenic properties that may cause transitional cell
carcinoma of the bladder
Cyclophosphamide (A) is
an alkylating agent, attaching an alkyl group to the guanine base of
DNA. This causes damage to the DNA structure and therefore prevents
cell replication
Adverse effects
include nephrotoxicity, hepatotoxicity, diarrhoea and pancreatitis.
Cyclosporine A
inhibits T-cell proliferation by binding to FKBP-1A. Its
advantage lies in its low nephrotoxicity in comparison to other immunosuppressive
agents.
Rapamycin (Sirolimus)
A 56-year-old man who is due to undergo a kidney transplant is seen by the
transplant surgeon. The surgeon decides the patient should be started on an
immunosuppressive agent before the surgery to prevent rejection of the organ.
He prescribes a monoclonal antibody directed at the IL-2 receptor. Which drug
has been prescribed?
A Basiliximab
B Abatacept
C Rituximab
D Natalizumab
E Tocilizumab
Immunosuppressive agents which are directed against cell surface
antigens primarily target cluster of differentiation (CD) molecules.
Basiliximab (A) is an antibody directed towards IL-2 receptor α chain
(CD25) which causes reduction in T-cell proliferation. It is used as prophylactic
treatment of allograft rejection, most commonly in patients
undergoing kidney transplant. Adverse effects include increased risk of
infection as well as a long-term risk of malignancy.
is a monoclonal antibody
against α4-integrin, an adhesion receptor which mediates the migration
of T cells from the circulation to target organs;
is used in the treatment of multiple sclerosis (reduced
T-cell migration to the central nervous system by influencing endothelial
cells expressing VCAM1) and Crohn’s disease (reduced interaction
of MADCAM1 and α4-integrin at sites of inflammation in the gastrointestinal
tract).
Natalizumab
a monoclonal IL-6 receptor antibody,
indicated in Castleman’s disease and rheumatoid arthritis. IL-6 is a proinflammatory
cytokine which promotes the immune response; inhibition
thereby reduces macrophage, neutrophil, T-cell and B-cell activation
is hepatotoxic and raises serum cholesterol; liver function
tests and cholesterol must be monitored regularly.
Tocilizumab
A 45-year-old woman who has been diagnosed with rheumatoid arthritis is seen
by a rheumatologist. The doctor wishes to start the patient on a fully humanized
TNF-α monoclonal antibody to prevent progression of the disease.
A Infliximab
B Adalimumab
C Etanercept
D Ustekinumab
E Denosumab
Immunosuppressive agents may be directed at specific cytokines to
modify the pathogenesis of certain disease processes. Adalimumab
(B) is a fully human monoclonal antibody to TNF-α. TNF-α has the
physiological role of inducing pro-inflammatory cytokines as well as
promoting leukocyte migration and endothelial adhesion. Adalimumab
has a large number of indications, including rheumatoid arthritis,
ankylosing spondylitis and Crohn’s disease
is a mouse–
human chimeric TNF-α antagonist indicated in similar conditions to adalimumab.
Infliximab
Toxicity may
result in reduced protection against infection from TB, hepatitis B virus
and hepatitis C virus, a lupus-like condition, demyelination and malignancy.
Infliximab
is an antibody to the p40 subunit of Il-12 and IL-23,
thereby preventing T-cell and natural-killer cell activation. It is used in
the treatment of psoriatic arthritis.
Ustekinumab
A 42-year-old man is referred to the rheumatology outpatient clinic. The patient
has been experiencing muscle and joint pain for the past month. On examination
a heliotrope rash is observed on the patient’s eyelids. Blood tests reveal the
patient has circulating anti-nuclear antibodies. Which immunofluorescence staining
pattern will be observed in this disease process?
A Homogeneous
B Nucleolar
C Speckled
D Peripheral
E Kinetoplast
Anti-nuclear antibodies (ANA) are directed at the cell nucleus and are
present in a number of rheumatic autoimmune diseases. Indirect immunofluorescence
is an immunological technique that can be used to help
determine the ANA in question. In this scenario, the patient has signs
and symptoms suggestive of dermatomyositis. Dermatomyositis is characterized
by the presence of anti-Jo-1 antibodies, which will demonstrate
a speckled (C) pattern on immunofluorescence
Dermatomyositis
(and polymyositis) are inflammatory diseases of the peripheral skeletal
muscles. The disease is associated with HLA DR3 and DR52. Clinical
features include weakness of the proximal muscles of the arms and legs;
on direct questioning there may be difficulty climbing stairs for example.
Dermatological manifestations include the presence of a heliotrope
on the eyelids and Gottron’s papules. Dermatomyositis is associated
with increased risk of lung, ovary, breast and stomach cancer. Other
antibodies which demonstrate a speckled appearance on immunofluorescence
include anti-Smith (SLE), anti-RNP (mixed connective tissue
disease) and anti-Ro (Sjögren’s disease).
A homogeneous pattern on indirect immunofluorescence
Consistent with anti-hisotne antibodies characteristic of drug induced SLE
Nucleolar pattern on indirect immunofluorescnece
Indicative of anti-RNA polymerase which suggests systemic sceloris
Peripheral pattern on indirect immunofluorescence
Found in the presence of anti-double stranded dsDNA antibodies in SLE
Kinetoplasts
Mitochondria found in Crithidialuciliae a non=pathological haemoflagellate and may be used as a substrate for pure dsDNA in the dx of SLE
A 54-year-old woman is referred to a rheumatologist. The patient states that she
has noticed her fingers becoming very pale on cold days; when she heats her hands against the radiator, she notices her hands becoming red. She mentions
that she has also had joint pains in her hands. On inspection, the patient has a
small mouth. Which of the following factors is most responsible for fibrosis in
this disease process?
A von Willebrand factor
B IL-2
C TGF-β
D TNF-α
E Endothelin-1
Systemic sclerosis is a chronic, inflammatory condition characterized
by fibrosis of the skin, blood vessels and internal organs. It can be
classified into a form that has major skin involvement (diffuse systemic
sclerosis) and a form in which skin involvement is limited to the distal
limbs and face (limited systemic sclerosis; CREST). CREST is defined by
calcinosis, Raynaud’s phenomenon, oesophageal dysmotility, sclerodactyly
and telangiectasia and is associated with the presence of circulating
anti-centromere antibodies. Given the absence of diffuse cutaneous
manifestations and combined with the symptoms and signs, the diagnosis
is limited systemic sclerosis. TNF-β (C) is central to the pathogenesis
of limited systemic sclerosis. Together with platelet-derived growth
factor (PDGF), TNF-β, produced by macrophages and T cells (IL-2 (B)
produced by CD4+ T cells induces further proliferation of T cells),
stimulate collagen production by fibroblasts. Collagen is deposited in
the extracellular matrix of the skin, oesophagus, alveoli of the lungs,
myocardium of the heart, liver and blood vessels; the pro-fibrotic state
correlates with the clinical features of limited systemic sclerosis.
A 12-year-old boy is referred to the paediatric endocrinology outpatient clinic
after experiencing recent onset weight loss, tiredness, frequency of urination and
thirst. A fasting plasma glucose test reveal a level of 10.1 mmol/L and a diagnosis
of type 1 diabetes mellitus is made. Which of the following autoantibodies
has tyrosine phosphatase as the target antigen?
A Islet cell surface antibody
B Insulin autoantibody
C Anti-glutamic acid decarboxylase antibody
D Anti-IA-2 antibody
E Islet cell antibody
Type 1 diabetes mellitus (T1DM) is a hyperglycaemic state caused by
autoimmune destruction of the β-cells in the islets of Langerhans of the
pancreas. The β-cells are responsible for the production of insulin. The
underlying pathogenesis of T1DM relates to T-cell mediated damage of
β-cells. The presence of glucose in the urine leads to the symptom of
polyuria (glucose is a potent osmolyte attracting water to enter the renal
tubules via osmosis). Polydipsia, weight loss and thirst are other characteristic
clinical features. An overnight fasting plasma glucose level of
above 7.0 mmol/L is diagnostic of diabetes. Another investigative test
which can be used is the oral glucose tolerance test. T1DM affects men
and women equally and usually presents in the pubertal years. T1DM is
strongly associated with HLA DR3 and DR4 alleles.
A number of autoantibodies are implicated in the disease process
of T1DM. In this case autoantibodies to tyrosine phosphatase have
been detected. Two antibodies to tyrosine phosphatase are present in
T1DM: anti-IA-2 antibodies (D) and anti-phogrin antibodies. Tyrosine
phosphatase autoantibodies are found in approximately 75 per cent
of patients with T1DM.
A 10-year-old boy is referred to a paediatrician after experiencing a seizure
1 week previously. Blood tests reveal that the seizure may have occurred
secondary
to low calcium levels; blood glucose levels are found to be high.
The child was already being investigated for ptosis and difficulty with eye
movements.
What is the most likely diagnosis?
A Hirata’s disease
B IPEX
C Kearns–Sayre syndrome
D POEMS syndrome
E APECED syndrome type 1
The autoimmune polyendocrine syndromes are a group of conditions
characterized by autoimmune disease affecting numerous endocrine
(and non-endocrine) organs. This child has symptoms, signs and investigative
features consistent with Kearns–Sayre syndrome (oculocraniosomatic
disease; C). Kearns–Sayre syndrome is a myopathic disease
caused by deletions of mitochondrial DNA. Initially, the disease process
affects the eyelid and extra-ocular muscles leading to ptosis and difficulty
with eye movement. Pigmentary retinopathy is another feature,
causing diffuse pigmentation of the retina. Other clinical manifestations
of Kearns–Sayre syndrome are proximal muscle weakness, cardiac conduction
defects, hearing loss and cerebellar ataxia. Endocrine system
effects include: hypoparathyroidism (causing hypocalcaemia), primary
gonadal failure, diabetes mellitus and hypopituitarism. Hirata’s disease
(insulin autoimmune syndrome; A), in contrast to Kearne–Sayre syndrome,
is defined by fasting hypoglycaemia as well as autoantibodies to
serum insulin. It is most prevalent in Japan (third most common cause
of hypoglycaemia) but extremely rare in other countries
The condition manifests with diabetes mellitus, eczema
and enteropathy.
IPEX
is the acronym given to the
following collection of clinical features: polyneuropathy/papilledoema/ pulmonary disease, organomegaly/oedema, endocrinopathy, M-protein
(usually IgG or IgM) and skin abnormalities (hyperpigmentation and
hypertrichosis).
POEMS syndrome
is associated with mild immune deficiency, dysfunctional
parathyroid gland/adrenal gland, hypothyroidism, gonadal
failure, alopecia and vitiligo) and results from mutations in the AIRE
gene, a key player in central tolerance.
APECED syndrome type 1 (autoimmune polyendocrine
syndrome type 1;
A 6-year-old girl presents to accident and emergency with severe haematemesis,
endoscopy revealing the presence of oesophageal varices. Blood tests reveal liver
function test derangement and a low level of circulating IgA. Subsequent liver
biopsy demonstrates interface hepatitis. Treatment with steroids shows a poor
response. Which autoantibody is most likely to be present in this child?
A Anti-nuclear antibody
B Anti-smooth muscle antibody
C Anti-liver kidney microsomal antibody
D Anti-mitochondrial antibody
E Anti-HBs antibody
This patient is most likely to have autoimmune hepatitis (AIH) given the
biopsy findings of interface hepatitis, which is typical of the disease.
AIH is a disease of unknown aetiology characterized by inflammation,
hepatocellular necrosis and fibrosis, which may ultimately lead to cirrhosis
and liver failure. Diagnosis is based on a combination of histological
and antibody evidence. Patients will commonly have a history
of other autoimmune disease. In this case, the patient is most likely to
have AIH type 2 due to the early age of diagnosis (more common in
paediatric population) and poor steroid response. AIH type 2 is characterized
by the presence of anti-liver kidney microsomal antibodies (C).
AIH type 2 also has an association with IgA deficiency. A diagnosis of
AIH type 1 is suggested by the presence of anti-nuclear antibodies (A)
and anti-smooth muscle antibodies (B). AIH type 2 may be diagnosed
in patients from 10 years of age to elderly patients. The disease course
is less severe than type 2 and responds well to steroid therapy. There is
also a third type of AIH which is characterized by the presence of antisoluble
liver antigen antibodies.
Patients with IgA deficiency are also at increased risk
of developing
Coeliac disease
Increases the difficulty of serological testing
Skin histology shows the
presence of acantholytic cells, which is defined as the separation of
keratinocytes caused by loss of intercellular cadherin connections.
Clinical features include blisters appearing in the mouth and skin, which
are very friable. Unaffected skin becomes increasingly fragile and exfoliation
of such areas occurs with light rubbing (Nikolsky sign positive).
High dose steroids (with or without immunosuppressive agents such as
azathioprine) is the mainstay treatment
Pemphigus vulgaris
characterized by immunological and histological
findings similar to pemphigus vulgaris. However, it is desmoglein
1 which is the target for autoantibodies, and the clinical course is
far less severe
Pemphigus foliaceous
caused by autoantibodies to type
VII collagen, which forms anchors between the layers of the skin; as a
result, bullae are usually induced by trauma. Dermatitis
Epidermolysis bullosa (
is associated with linear IgA bullous
dermatosis (LABD), characterized by linear deposition of IgA on direct
immunofluorescence
Vancomycin
is a bullous disorder associated
with pregnancy (C). Bullae appear in the second or third trimester
of pregnancy, which are characterized by itchiness; the condition tends
to resolve post-partum
Pemphigus gestationis
Nonproliferative vs proliferative glomerulonephritis
Nonproliferative= nephrotic
Proliferatie= nephritic more commonly
Immunofluorescence will reveal the presence of IgM and C3 deposition
in affected areas. Patients will usually present with some degree of renal
impairment.
Focal segmental glomerulonephritis
may be idiopathic or occur secondary to conditions
such as Alport syndrome (A) and reflux nephropathy (B). Alport syndrome
is a hereditary syndrome (mutation of α4 chain of type IV collagen)
associated with glomerulonephritis, end-stage kidney disease and
hearing loss. Reflux nephropathy results from vesico-ureteric reflux due
to chronic pyelonephritis.
most commonly occurs in adults,
and demonstrates a thickened glomerular basement membrane and
spike/dome protrusions on histology. Direct immunofluorescence
reveals the presence of sub-epithelial granular deposits of IgG and
C3. Causes
Membranous glomerulonephritis
Patients with hydrocephalus who have a
cerebral shunt in situ are prone to
The pathogenesis involves the
increased risk of long-term bacterial infection, leading to immune
complex deposition in the glomeruli.
Shunt nephritis, a cause of membranous glomerulonephritis
Definition and pathogenesis of nephritis
Proliferative glomerulonephritides is characterized by an increased
number of cells in the glomerulus. This group of diseases usually
present with nephritic syndrome, defined by the presence of haematuria,
red cell casts, dysmorphic red cells, oliguria and hypertension
Proteinuria and oedema may also be present. Immune damage to
the glomerular vessels results in severe inflammation, allowing red
cells to pass into the tubule; in the process these red cells experience
mechanical damage while passing through the inflamed vessels and
as a result are dysmorphic. Cells of the distal convoluted tubule and
collecting duct secrete a glycoprotein called Tamm–Horsefell protein
which sticks red cells together forming cylindrical red cell casts
is defined by mesangial
cell proliferation with thickening of the capillaries. Two types exist:
type 1 in which there is classical and alternative complement pathway
activation and type 2 which is associated with only alternative pathway
activation.
Membranoproliferative glomerulonephritis
proliferation of
macrophages and parietal epithelial cells).
demonstrate the crescent sign on biopsy
Classification of Lupus Nephritis
Stage 1
Minimal mesangial LN
No changes on light microsocpy
Mesangial immune deposits
Classifciation of LN
Stage 2
Mesangial proliferative LN
Changes confined to mesangium
Mesangial immune deposits
Classification of LN
Stage 3
Focal lupus nephritis
Focal, segemental or glomerulonephriits involving <50% of all glomeruli
Subendothelial and mesangial deposits
Classification of LN
Stage 4
Focal, segmental or glomerulonephritis involving >50 of all glomeruli
Subendoethlial immune deposits
Classification of LN
Stage V
Membranous LN
Glomerular sclerosis involving >90% of glomeruli, fibrosis and tubular atrophy
Subepithelial and intramembranous immune deposits
No changes to glomeruli Mesangial immune deposits
Stage I (minimal mesangial lupus nephritis
Changes confined to
mesangium
Mesangial immune deposits
Stage II (mesangial proliferative lupus nephritis
Focal, segmental or
glomerulonephritis involving
<50 per cent of all glomeruli
Subendothelial and mesangial
immune deposits
Stage III (focal lupus nephritis)
Focal, segmental or
glomerulonephritis involving
>50 per cent of all glomeruli
Subendothelial immune deposits
Stage IV (diffuse proliferative nephritis)
Glomerular sclerosis involving
>90 per cent of glomeruli,
fibrosis and tubular atrophy
Subepithelial and
intramembranous immune
deposits
Stage V (membranous lupus nephritis)
defined by the presence of cryoglobulins in
the circulation; these are immunoglobulins that precipitate at low
temperatures. Secondary causes include connective tissue diseases and
lymphoproliferative conditions. It is, however, unknown why such
immunoglobulins are formed in the first instance. When precipitation
does occur at cold temperatures, the immunoglobulins adhere to vessel
walls, leading to complement activation, neutrophil recruitment and,
consequently, vessel damage.
Cryoglobulinaemia
A 35-year-old builder is referred to a neurologist after experiencing increasing
axial rigidity over the previous few weeks; his symptoms are interfering
with his work. The patient has a history of type 1 diabetes mellitus and vitiligo.
Immunological investigations reveal the presence of circulating anti-glutamic
acid decarboxylase antibodies. What is the most likely diagnosis?
A Myasthenia gravis
B Multiple sclerosis
C Acute disseminated encephalomyelitis
D Lambert–Eaton myasthenic syndrome
E Stiff man syndrome
The patient in question has presented with axial rigidity/stiffness associated
with a history of autoimmune disease and circulating anti-glutamic
acid decarboxylase antibodies (anti-GAD), which point to stiff man
syndrome (SMS; E) as the correct answer. SMS is a very rare neurological
condition which is poorly understood. Clinical features include
progressive axial and abdominal wall stiffness. It is strongly associated
with the presence of anti-GAD antibodies. However, only a small
minority of type 1 diabetes mellitus patients suffer with SMS, suggesting
that anti-GAD antibodies do not tell the whole story in terms of
aetiology. However, SMS does occur in patients who suffer from other
autoimmune
diseases including thyroid disease, pernicious anaemia
and type 1 diabetes mellitus
is defined by proximal muscle weakness,
which is improved by muscle contraction, loss of tendon reflexes
and autonomic nervous system dysfunction. Leg involvement is greater
than that of myasthenia gravis. It is considered a paraneoplastic syndrome
due to its association with small cell lung cancer.
caused
by autoantibodies that target the voltage-gated calcium channels of the
pre-synaptic membrane.
Lambert–Eaton
myasthenic syndrome
is a
demyelinating condition that follows vaccination or infection. Clinical
features include fever, headache and reduced consciousness; focal signs
include optic neuritis, cranial nerve palsies and seizures. Most cases are
followed by recovery within a few months.
Acute disseminated encephalomyelitis
A 35-year-old man is transferred to the intensive care unit for ventilator support
after suffering an episode of respiratory distress. The patient was admitted 5 days
previously after experiencing weakness of his legs. Approximately 2 weeks prior
to his admission the man had suffered a bout of gastroenteritis caused by the
bacterium Campylobacter jejuni. Which of the following is the most likely antigenic
target for autoantibodies in this disease process?
A Ganglioside LM1
B Ganglioside GM1
C Hu
D Myelin-associated glycoprotein
E Purkinje cells
Several polyneuropathies have an underlying immune component,
characterized by the presence of autoantibodies targeted at components
of the nervous system. In this scenario, the patient has experienced
weakness following a gastrointestinal infection, now complicated by
respiratory involvement. The most likely diagnosis is Guillain–Barrè
syndrome (GBS), for which ganglioside LM1(A) is the implicated target
for autoantibodies. GBS is a symmetrical inflammatory polyneuropathy
that begins in the legs and ascends to involve motor neurons of the
arms, face and finally those supplying muscles of respiration. GBS usually
follows an infection, most frequently after viral infection such as
cytomegalovirus or bacterial gastroenteritis caused by Campylobacter
jejuni. The pathogenesis involves cross-reactivity between antibodies
against the pathogen and components of peripheral nerve myelin
components, such as ganglioside LM1. Other potential myelin targets
include P2 protein and galactocerebroside.
suggested to be due to antibodies to
the ganglioside GM
Amyotrophic lateral sclerosis
(ALS) is a sub-type of motor neuron disease characterized by loss
of neurons in the motor cortex as well as anterior horn of the spinal cord; it is therefore associated with both upper and lower motor signs.
The pathogenesis of ALS has been suggested to be due to antibodies to
the ganglioside GM1 (
Antibodies are directed at Hu
Paraneoplastic subacute sensory neuropathy
(PSSN) is associated with malignancies such as small cell lung cancer.
Antibodies are directed at Hu (C) proteins which are a constituent part
of peripheral nerves.
is associated
with antibodies to Purkinje cells (E) of the central nervous system.
The pathogenesis is thought to be secondary to cross-reactivity between
antibodies to tumour cells and antigens present on cerebellar Purkinje
cells
Paraneoplastic cerebellar degeneration
typified by the presence
of antibodies that target myelin-associated glycoprotein
Paraprotein-associated polyneuropathy
A 35-year-old woman is referred to an ophthalmologist after seeing floaters in
her right eye. On examination, there is loss of accommodation in the same eye.
The patient’s notes reveal there had been trauma to the left eye following a car
accident 3 weeks previously. It is explained to the patient that she could suffer
potential loss of vision if steroid treatment is not commenced urgently. What is
the most likely diagnosis?
A Keratoconjunctivitis sicca
B Sympathetic ophthalmia
C Uveitis
D Keratitis
E Scleritis
Immune disorders of the eye can be classified according to the anatomical
site of disease: cornea, sclera/episclera, uvea and retina. This
patient presents with floaters and loss of accommodation in her right
eye, several weeks after experiencing trauma to her left eye. The most
likely diagnosis is therefore sympathetic ophthalmia (B), a granulomatous
CD4+ T-cell mediated disease. The trigger for the disease is trauma
to the damaged eye. The eye is an immunoprivileged site and is therefore,
under normal circumstances, protected from possible autoimmune
attack. Trauma to the eye breaks such tolerance, and there is consequently
increased photoreceptor antigen presentation to immune cells,
triggering cytokine release and recruitment of CD4+ T cells. These CD4+
cells soon encounter the same antigen presented at normal levels in
the healthy eye, leading to a break in tolerance. Activated T cells cause
ocular damage which may, in severe cases, lead to blindness.
this
phenomenon usually occurs after an infection by mycoplasma or EBV.
Cold AIHA
causes include lymphoproliferative disorders,
drugs (penicillin) and autoimmune diseases (SLE
Warm AIHA